Вы находитесь на странице: 1из 45

DAVAO SAW MILL CO., INC., plaintiff-appellant, vs.APRONIANO G.

CASTILLO and DAVAO


LIGHT & POWER CO., INC.,defendants-appellees.
G.R. No. L-40411
August 7, 1935
MALCOLM, J.:

Facts:
Davao Saw Mill Co., Inc., is the holder of a lumber concession from the Government of the
Philippine Islands. However, the land upon which the business was conducted belonged to
another person. On the land the sawmill company erected a building that housed the
machinery used by it. Some of the implements thus used were clearly personal property, the
conflict concerning machines, which were placed and mounted on foundations of cement. In
the contract of lease between the sawmill company and the owner of the land there appeared
the following provision: That on the expiration of the period agreed upon, all the improvements
and buildings introduced and erected by the party of the second part shall pass to the exclusive
ownership of the lessor without any obligation on its part to pay any amount for said
improvements and buildings; which do not include the machineries and accessories in the
improvements. In another action wherein the Davao Light & Power Co., Inc., was the plaintiff
and the Davao, Saw, Mill Co., Inc., was the defendant, a judgment was rendered in favor of the
plaintiff in that action against the defendant; a writ of execution issued thereon, and the
properties now in question were levied upon as personality by the sheriff. No third party claim
was filed for such properties at the time of the sales thereof as is borne out by the record made
by the plaintiff herein It must be noted also that on number of occasion, Davao Sawmill
treated the machinery as personal property by executing chattel mortgages in favor of third
persons. One of such is the appellee by assignment from the original mortgages. The lower
court rendered decision in favor of the defendants herein. Hence, this instant appeal.
Issue:
Whether or not the machineries and equipment were personal in nature.
Held:Yes. The Supreme Court affirmed the decision of the lower court. Machinery, which is
movable in its nature, only becomes immobilized when placed in a plant by the owner of the
property or plant, but not when so placed by a tenant, a usufructuary, or any person having
only a temporary right, unless such person acted as the agent of the owner.
Lopez vs. Orosa, Jr. and Plaza Theatre, Inc.
G.R. Nos. L-10817-18.
February 28, 1958.
Felix, J.
Facts:

Sometime in May, 1946, Vicente Orosa, Jr., invited Lopez to make an investment in the theatre
business. Although Lopez expressed his unwillingness to invest of the same, he agreed to supply
the lumber necessary for the construction of the proposed theatre, and at Orosas request and
assurance that the latter would be personally liable for any account that the said construction
might incur, Lopez further agreed that payment therefore would be on demand and not cash
on delivery basis. With this, Lopez delivered the lumber which was used for the construction of
the Plaza Theatre on May 17, 1946, up to December 4 of the same year. The total cost of
materials amounted to P62,255.85 but Lopez was only paid P20,848.50, thus leaving a balance
of P41,771.35. Orosa and Rustia, corporation president, promised Lopez to obtain a bank loan
to satisfy the balance, to which assurance Lopez had to accede. Unknown to Lopez, Orosa and
Rustia already secured a loan for P30,000 from the PNB with the Luzon Surety Company as
surety, and the corporation in turn executed a mortgage on the land and building in favor of
said company as counter-security. As the land at that time was not yet brought under the
operation of the Torrens System, the mortgage on the same was registered on 16 November
1946, under Act 3344. Subsequently, when the corporation applied for the registration of the
land under Act 496, such mortgage was not revealed and thus OCT O-391 was correspondingly
issued on October 25, 1947, without any encumbrance appearing thereon.
Persistent demand from Lopez caused Vicente Orosa, Jr. to execute, on 17 March 1947, an
alleged deed of assignment of his 420 shares of stock of the Plaza Theater, Inc., at P100 per
share or with a total value of P42,000 in favor of the creditor, and as the obligation still
remained unsettled, Lopez filed on 12 November 1947, a complaint with the CFI Batangas
against Vicente Orosa Jr. and Plaza Theatre, Inc., praying that defendants be sentenced to pay
him jointly and severally the sum of P41,771.35 with legal interest from the filing of the action;
that in case defendants fail to pay the same, that the building and the land owned by the
corporation be sold at public auction and the proceeds thereof be applied to said indebtedness.
Plaintiff also caused the annotation of a notice of lis pendens on said properties with the
Register of Deeds.
The surety company upon discovery that the land was already registered under the Torrens
System and that there was a notice of lis pendens thereon, filed a petition for review of the
decree of the land registration court in order to annotate the lights and interests of the surety
company over said properties. Lopez opposed by asserting that the amount demanded by him
constituted a preferred lien over the properties of the obligors; that the surety company was
guilty of negligence when it failed to present an opposition to the application for registration of
the property; and that if any annotation of the rights and interest of said surety would ever be
made, same must be subject to the lien in his favor. The court ruled that Orosa and the Plaza
Theatre, Inc., were jointly liable for the unpaid balance of the cost of lumber used in the
construction of the building and the plaintiff thus acquired the material mans lien over the
same; the lien being merely confined to the building and did not extend to the land on which
the construction was made.
Issue: Whether material mans lien for the value of the materials used in the construction of a
building attaches to the building alone and does not extend to the land on which the building is
adhered to.

Held: Yes, the lien is limited to the theater to which it was used only and does not extend to
the land. The material mans lien could be charged only to the building for which the credit was
made or which received the benefit of refection, the lower court was right in, holding at the
interest of the mortgagee over the land is superior and cannot be made subject to the material
man's lien. Real estate connotes the land and the building constructed. The inclusion of the
building in the enumeration of what may constitute real properties could only mean one
thingthat a building is by itself an immovable property. The law gives preference to
unregistered refectionary credits only with respect to the real estate upon which the
refectionary or work was made. In the absence of any specific provision of law to the contrary,
a building is an immovable property, irrespective of whether or not said structure and the land
on which it is adhered to belong to the same owner.
Title: Fels Energy Inc. v The Province of Batangas
G.R. No.: 168557
Date: February 16, 2007
Ponente: Callejo, Sr.
FACTS: National Power Corporation(NPC) entered into a lease contract with Polar Energy, Inc.
over diesel engine power barges moored at Balayan Bay in Calaca, Batangas. It was stipulated
in the contract that NPC shall be responsible for the payment of taxes to wit:
10.1 RESPONSIBILITY. NAPOCOR shall be responsible for the payment of (a) all taxes, import
duties, fees, charges and other levies imposed by the National Government of the Republic of
the Philippines or any agency or instrumentality thereof to which POLAR may be or become
subject to or in relation to the performance of their obligations under this agreement (other
than (i) taxes imposed or calculated on the basis of the net income of POLAR and Personal
Income Taxes of its employees and (ii) construction permit fees, environmental permit fees and
other similar fees and charges) and (b) all real estate taxes and assessments, rates and other
charges in respect of the Power Barges.
Subsequently, Polar Energy, Inc. assigned its rights under the Agreement to FELS. FELS then
received an assessment of real property taxes on the power barges from Provincial Assessor of
Batangas City. FELS referred the matter to NPC, reminding it of its obligation under the
Agreement to pay all real estate taxes. It then gave NPC the full power and authority to
represent it in any conference regarding the real property assessment of the Provincial
Assessor. NPC sought reconsideration of the Provincial Assessors decision to assess real
property taxes on the power barges. However, the motion was denied and the Provincial
Assessor advised NPC to pay the assessment. This prompted NPC to file a petition with the
Local Board of Assessment Appeals (LBAA) for the setting aside of the assessment and the
declaration of the barges as non-taxable items; it also prayed that should LBAA find the barges
to be taxable, the Provincial Assessor be directed to make the necessary corrections. In its
Answer to the petition, the Provincial Assessor averred that the barges were real property for
purposes of taxation under Section 199(c) of Republic Act (R.A.) No. 7160.
The LBAA ruled that the power plant facilities, while they may be classified as movable or
personal property, are nevertheless considered real property for taxation purposes because

they are installed at a specific location with a character of permanency. The LBAA also pointed
out that the owner of the bargesFELS, a private corporationis the one being taxed, not NPC. A
mere agreement making NPC responsible for the payment of all real estate taxes and
assessments will not justify the exemption of FELS; such a privilege can only be granted to NPC
and cannot be extended to FELS. Finally, the LBAA also ruled that the petition was filed out of
time.
ISSUE/S: WON power barges, which are floating and movable, are real properties and
therefore, subject to real property tax.
HELD: Yes. The court held that as found by the appellate court, the CBAA and LBAA, power
barges are real property and are thus subject to real property tax. Article 415 (9) of the New
Civil Code provides that "docks and structures which, though floating, are intended by their
nature and object to remain at a fixed place on a river, lake, or coast" are considered
immovable property. Thus, power barges are categorized as immovable property by
destination, being in the nature of machinery and other implements intended by the owner for
an industry or work which may be carried on in a building or on a piece of land and which tend
directly to meet the needs of said industry or work.
The Court affirm the findings of the LBAA and CBAA that the owner of the taxable properties is
petitioner FELS, which in fine, is the entity being taxed by the local government.
Benitez vs. Court of Appeals
266 SCRA 242
G.R. No. 104828
January 16, 1997
FACTS:
In July 1989, the spouses Renato and Elizabeth Macapagal purchased a parcel of land
adjoining the lot of spouses Rafael and Avelina Benitez. After they conducted a survey on said
land, the spouses Macapagal discovered that the house of spouses Benitez was already
encroaching their property. Macapagal then demanded in writing for Benitez to vacate.
Eventually, an ejectment suit was filed by Macapagal against Benitez.
The trial court ruled in favor of Macapagal as it found Benitez to be a builder in bad
faith. On appeal, Benitez insists that they are builders in good faith hence he should be given
the option to exercise the right of pre-emption (the right to buy the said portion of land owned
by Macapagal which is being occupied by his [Benitez] house.) On appeal, the Regional Trial
Court of Pasig, Branch 167, affirmed said decision. The RTC said: "The controversy in this case is
not an encroachment or overlapping of two (2) adjacent properties owned by the parties. It is a
case where a part of the house of the defendants is constructed on a portion of the property of
the plaintiffs. So that as new owner of the real property, who has a right to the full enjoyment
and possession of the entire parcel covered by Transfer Certificate of Title No. 41961, plaintiffs
have the right to demand that defendants remove the portion of the house standing on
plaintiff's realty. . . ."

ISSUE: Whether or not the right of pre-emption may be exercised by a builder in good faith.
HELD:
No. In the first place, there is already a factual finding by the trial court that Benitez was
a builder in bad faith.
Secondly, even assuming that Benitez was a builder in good faith, he cannot exercise
said right of pre-emption. The right of pre-emption or the right to sell is solely lodged in the
owner in good faith as provided for by Article 448 of the Civil Code. This advantage in Article
448 is accorded the landowner in good faith because his right is older, and because, by the
principle of accession, he is entitled to the ownership of the accessory thing. Otherwise stated,
the builder in good faith has no pre-emptive right to buy even as a compromise, as this
prerogative belongs solely to the landowner in good faith.
The right is not shifted in favor of the builder in bad faith. And this situation is now governed by
Article 450 of the Civil Code which gives the owner in good faith the options either:
a. to demand the builder in bad faith to demolish what he built; or
b. to compel the builder in bad faith to pay the price of the land.
PEDRO P. PECSON, petitioner, vs. COURT OF APPEALS, SPOUSES JUAN NUGUID and ERLINDA
NUGUID, respondents.
Facts:
Petitioner Pedro Pecson was the owner of a commercial lot in Quezon City, on
which he built a four-door two-storey apartment building. For his failure to pay realty taxes, the
lot was sold at public auction to Mamerto Nepomuceno who in turn sold it to private
respondents. The petitioner challenged the validity of the auction sale. However, the RTC
dismissed the complaint, but as to the private respondents' claim that the sale included the
apartment building, it held that the issue concerning it was "not a subject of the litigation". The
private respondents filed with the trial court a motion for delivery possession of the lot and the
apartment building, citing Art. 546 of the New Civil Code. The trial court ruled that the movant
shall reimburse plaintiff the construction cost and that movant Juan Nuguid is entitled to
immediate issuance of a writ of possession over the subject lot and improvements. The
petitioner moved for the reconsideration of the order but it was not acted upon by the trial
court. Instead, it issued a writ of possession directing the deputy sheriff "to place said movant
Juan Nuguid in possession of subject property with all the improvements and eject therefrom
all occupants therein, their agents, assignees, heirs, and representatives.
Issue:
Whether or not Art. 448 shall apply in a case where the owner of the land is the
builder who then loses ownership of the land by sale or public auction.
Held:
No, Art. 448 does not apply in a case where the owner of the land is the builder
who then loses ownership of the land by sale or public auction.
The Supreme Court ruled that under Art. 448 of the Civil Code, the owner of the land on which

anything had been built, sown, or planted in good faith, shall have the right to appropriate as
his own the works, sowing, or planting, after payment of the indemnity provided for in Art. 546
and 548, or to oblige the one who built or planted to pay the price of the land and the one who
sowed, the proper rent. However, the builder or planter cannot be obliged to buy the land if its
value is considerably more than of the building or trees.
In such a case, he shall pay reasonable rent, if the owner of the land does not choose to
appropriate the building or trees after proper indemnity. The parties shall agree upon the terms
of the lease and in case of disagreement, the court shall fix the terms thereof.
By its clear language, Art. 448 refers to a land whose ownership is claimed by two or more
parties, one of whom has built some works, or sown or planted something. The building, sowing
or planting may have been made in good faith or in bad faith. The rule on good faith laid down
in Art. 546 of the NCC shall be applied in determining whether a builder, sower, or planter had
acted in good faith.
Thus, in strict point of law, Art. 448 is nit apposite to the case at bar. Nevertheless, we believe
that the provision therein on indemnity may be applied by analogy considering that the primary
intent of Art. 448 is to avoid a state of forced co-ownership and that the parties, including the
two courts below, in the main agree that Arts. 448 and 546 of the New Civil Code are applicable
and indemnity of the improvements may be paid although they differ as to the basis of the
indemnity
Title: SPOUSES JUAN NUGUID AND ERLINDA T. NUGUID vs CA AND PEDRO P. PECSON
G.R. No.: G.R. No. 151815
Date: February 23, 2005
Ponente: QUISUMBING

FACTS:
Pedro P. Pecson owned a commercial lot on which he built a four-door two-storey apartment
building. For failure to pay realty taxes, the lot was sold at public auction by the City Treasurer
to Mamerto Nepomuceno, who in turn sold it for 103,000 pesos to the spouses Juan and
Erlinda Nuguid. Pecson challenged the validity of the auction sale before the RTC of Quezon
City, the RTC upheld the spouses title but declared that the four-door two-storey apartment
building was not included in the auction sale. This was affirmed by the CA and by the Supreme
Court. The Nuguids became the uncontested owners of commercial lot. The Nuguid spouses
moved for delivery of possession of the lot and the apartment building.
ISSUE:
Whether spouses Nuguid should reimburse Pecson for the benefits derived from the apartment
building?

HELD:
Yes. Spouses Nuguid opted to appropriate the improvement for themselves as early as June
1993, when they applied for a writ of execution despite knowledge that the auction sale did not
include the apartment building, they could not benefit from the lots improvement, until they
reimbursed the improver in full, based on the current market value of the property.
Under Article 448, the landowner is given the option, either to appropriate the improvement as
his own upon payment of the proper amount of indemnity or to sell the land to the possessor in
good faith. Relatedly, Article 546 provides that a builder in good faith is entitled to full
reimbursement for all the necessary and useful expenses incurred; it also gives him right of
retention until full reimbursement is made.
The right of retention is considered as one of the measures devised by the law for the
protection of builders in good faith. Its object is to guarantee full and prompt reimbursement as
it permits the actual possessor to remain in possession while he has not been reimbursed (by
the person who defeated him in the case for possession of the property) for those necessary
expenses and useful improvements made by him on the thing possessed.
Given the circumstances of the instant case where the builder in good faith has been clearly
denied his right of retention for almost half a decade, we find that the increased award of
rentals by the RTC was reasonable and equitable. The petitioners had reaped all the benefits
from the improvement introduced by the respondent during said period, without paying any
amount to the latter as reimbursement for his construction costs and expenses. They should
account and pay for such benefits.
TITLE: TECHNOGAS PHILIPPINES vs. CA
G.R. No. 108894
DATE: February 10, 1997
PONENTE: PANGANIBAN, J.
FACTS: The parties in this case are owners of adjoining lots in Paraaque, Metro Manila. It was
discovered in a survey, that a portion of a building of Technogas, which was presumably
constructed by its predecessor-in-interest, encroached on a portion of the lot owned by private
respondent Edward Uy. Upon learning of the encroachment or occupation by its buildings and
wall of a portion of private respondents land, the petitioner offered to buy from defendant
that particular portion of Uys land occupied by portions of its buildings and wall with an area of
770 square meters, more or less, but the latter, however, refused the offer. The parties entered
into a private agreement before a certain Col.
Rosales in Malacaang, wherein petitioner agreed to demolish the wall at the back portion of
its land thus giving to the private respondent possession of a portion of his land previously
enclosed by petitioner's wall. Uy later filed a complaint before the office of Municipal Engineer
of Paraaque, Metro Manila as well as before the Office of the Provincial Fiscal of Rizal
against Technogas in connection with the encroachment or occupation by plaintiff's buildings
and walls of a portion of its land but said complaint did not prosper; so Uy dug or caused to be

dug a canal along Technogas wall, a portion of which collapsed in June, 1980, and led to
the filing by the petitioner of the supplemental complaint in the above-entitled case and
a separate criminal complaint for malicious mischief against Uy and his wife which
ultimately resulted into the conviction in court Uy's wife for the crime of malicious mischief.
ISSUE: WON the petitioner is builder in good faith.
HELD: Yes. Article 527 of the Civil Code presumes good faith, and since no proof exists to
show that the encroachment over a narrow, needle-shaped portion of private respondent's
land was done in bad faith by the builder of the encroaching structures, the latter should be
presumed to have built them in good faith. It is presumed that possession continues to be
enjoyed in the same character in which it was acquired, until the contrary is proved. Good faith
consists in the belief of the builder that the land he is building on is his, and his ignorance of any
defect or flaw in his title. Hence, such good faith, by law, passed on to Pariz's successor,
petitioner in this case. The good faith ceases from the
moment defects in the title are made known to the possessor, by extraneous evidence or by
suit for recovery of the property by the true owner. Consequently, the builder, if sued by the
aggrieved landowner for recovery of possession, could have invoked the provisions of Art. 448
of the Civil Code. The obvious benefit to the builder under this article is that, instead of
being outrightly ejected from the land, he can compel the landowner to make a choice between
the two options: (1) to appropriate the building by paying the indemnity required by law, or (2)
sell the land to the builder. The landowner cannot refuse to exercise either option and
compel instead the owner of the
building to remove it from the land. In view of the good faith of both petitioner and private
respondent, their rights and obligations are to be governed by Art. 448. He cannot exercise a
remedy of his own liking

Title: Rosales v Castelltort


G.R. No.: 157044
Date: 5 October 2005
Ponente: Carpio-Morales, J.
FACTS:
Petitioners were the registered owners of Lot 17 of a subdivision in Los Baos, Laguna.
Respondents were the registered owners of Lot 16, adjoining it.
In August 1995, the Sps. Rosales discovered that Mr. Castelltort had started to build a house on
Lot 17. When he was notified, Castelltort explained that he built his house on that lot only
because a geodetic engineer had surveyed the land and pointed to Lot 17 as the Lot 16 that he
and his wife purchased.
Sps. Rosales were offered, but refused, both a substitute lot in the same subdivision and
payment of Lot 17s purchase price. Petitioners then, through counsel, directed Castelltort to

stop construction of and to demolish his house. Subsequently, they filed suit against Sps.
Castelltort.
RTC ruled for petitioners, holding that Sps. Castelltort were not builders in good faith. CA
reversed. Hence this petition.
ISSUE: W/N Castelltort is a builder in good faith
HELD: Castelltort is a builder in good faith.
1. Art. 527, Civil Code provides that good faith is presumed. In this case, the presumption
holds because no bad faith could be imputed to Castelltort:
Prior to the purchase of Lot 16, because the seller could only produce a photocopied
TCT, Castelltort acted prudently and obtained from the Register of Deeds a certified
true copy of that TCT.
Castelltort only built on Lot 17 because the seller informed him that it was Lot 16.
The seller in turn relied on the geodetic engineers identification of the lot, which
was mistaken due to his teams inaccurate determination of a reference point in the
subdivision. Seller further pointed out that Lots 17 and 16 look practically identical.
2. For this reason, Sps. Rosales can only choose between the two options available to the
landowner under Art. 448, Civil Code, i.e., (a) appropriating the improvement, or (b)
compelling the builder to buy the land occupied by the improvement. They cannot at
the outset compel the builder in good faith in this case, Castelltort to remove his
building.
3. Castelltorts good faith ceased when he learned of the defect in his possession (Art.
528, Civil Code). Whether the Sps. Rosales choose appropriation or forced sale, the
calculation of reasonable rent to be paid by Castelltort must use that as the starting
point of accrual of rent.
9.G.R. No. 117642 April 24, 1998EDITHA ALVIOLA and PORFERIO ALVIOLA, petitioners,vs.
HONORABLE COURT OF APPEALS, FORENCIA BULING Vda de TINAGAN, DEMOSTHENES
TINAGAN,
JESUS
TINAGAN,ZENAIDA
T.,
JOSEP
and
JOSEPHINE
TINAGAN,
respondents.
Facts:. On April 1, 1950, Victoria Tinagan purchased from Mauro Tinagan 2 parcels of land. One
parcel of land contains an area of 5,704 square meters, more or less; while the other contains
10,860 square meters. Thereafter, Victoria and hser son Agustin, took possession of said parcels
of land. Sometime in 1960, petitioners occupied portions thereof whereat they built a copra
dryer and put up a store wherein they engaged in the business of buying and selling copra. On
June 23, 1975, Victoria died. On October 26, 1975, Agustin died, survived by herein private
respondents. On December 24, 1976, petitioner Editha assisted by her husband filed a
complaint for partition and damages, claiming to be an acknowledged natural child of deceased
Agustin Tinagan and demanding the delivery of her shares in the properties left by the
deceased.

This case was dismissed by the trial court on the ground that recognition of natural children
may be brought only during the lifetime of the presumed parent and petitioner Editha did not
fall in any of the exceptions enumerated in Article 285 of the Civil Code.
On March 29, 1988, private respondents filed a complaint for recovery of possession against
Editha and her husband Porferio, praying, among others, that they be declared absolute owners
of the said parcels of land, and that petitioners be ordered to vacate the same, to remove their
copra dryer and store, to pay actual damages (in the form of rentals),moral and punitive
damages, litigation expenses and attorney's fees. In their answer, petitioners contend that they
own the improvements in the disputed properties which are still public land; that they are
qualified to be beneficiaries of the comprehensive agrarian reform program and that they are
rightful possessors by occupation of the said properties for more than twenty years. After trial,
the lower court rendered judgment in favor of the private respondents. CA affirmed. Hence,
thispetition.
Issue: W/N the lands in question are public lands and W/N the petitioners are rightful
possessors
by
occupation
for
more
than
20
year.
Held: Petitioners claim that the disputed properties are public lands. This is a factual
issue. The private respondents adduced overwhelming evidence to prove their ownership and
possession of the two (2) parcels of land on portions of which petitioners built the copra dryer
and a store. Private respondents tax declarations and receipts of payment of real estate taxes,
as well as other related documents, prove their ownership of the disputed properties. As
correctly ruled by the respondent court, there was bad faith on the part of the petitioners when
they constructed the copra dryer and store on the disputed portions since they were fully
aware that the parcels of land belonged to Victoria Tinagan. And, there was likewise bad faith
on the part of the private respondents, having knowledge of the arrangement between
petitioners and Victoria Tinagan relative to the construction of the copra dryer and store. Thus,
for purposes of indemnity, Article 448 of the New Civil Code should be applied. [32] However, the
copra dryer and the store, as determined by the trial court and respondent court, are
transferable in nature. Thus, it would not fall within the coverage of Article 448. As the noted
civil law authority, Senator Arturo Tolentino, aptly explains: To fall within the provision of this
Article, the construction must be of permanent character, attached to the soil with an idea of
perpetuity; but if it is of a transitory character or is transferable, there is no accession, and the
builder must remove the construction. The proper remedy of the landowner is an action to
eject the builder from the land.[33]
The private respondents action for recovery of possession was the suitable solution to eject
petitioners from the premises.
Title: ADELFO MACEDA vs. HON. COURT OF APPEALS AND CEMENT CENTER, INC.
G.R. No.: 83545
Date: August 11, 1989
Ponente: Justice Grio-Aquino

FACTS: The leased property originally belonged to the spouses Arturo Victoria and Maxima
Monserrat, a maternal aunt of the petitioner. After the spouses migrated to the U.S. in 1970,
they leased their house and lot in San Juan, Metro Manila. As the house was old and run down,
petitioner proposed to have it repaired and renovated, subject to reimbursement of his
expenses.
Upon the death of the spouses the property was sold to Zubiri then was sold again to Cement
Center, Inc., which obtained title to the property. The president of the company inspected the
premises. Maceda was asked to vacate the property because the company would build a
housing project on it for its employees. Maceda insisted on being reimbursed for his
improvements as the original owners had promised to do so. Formal demands to vacate for
payment of P4,000 monthly rental from April 15, 1982 were sent to him by the company. An
ejectment suit was filed against him in the Metropolitan Trial Court of San Juan, Metro Manila.
In this answer to the complaint, Maceda set up a counterclaim, for 240,000, which is the
alleged value of his improvements.
In its decision MTC ordered him to vacate the premises and pay the plaintiff P2,000 per month
as reasonable compensation for his use of the premises until he actually vacates, and P5,000 as
attorney's fees. It ordered plaintiff to pay the defendant P158,000 as the value of his
improvements and repairs, less his accrued rentals of P64,000 as a December 1985 and the sum
of P12,000 which he had earlier received as partial reimbursement.
Both parties appealed to the Regional Trial Court. The Regional Trial Court set aside the inferior
court's decision. On May 19, 1987, it dismissed the ejectment complaint, and ordered Cement
Center to pay Maceda P182,000 for his necessary and useful improvements. Cement Center
filed a petition for review in the Court of Appeals. Court of Appeals rendered a decision,
modifying the appealed decision, the dispositive part of which reads thus:
"PREMISES CONSIDERED, the decision appealed from is hereby AFFIRMED insofar as it
dismissed the complaint for ejectment filed by petitioner against private respondent. However,
the portions of the decision declaring petitioner(plaintiff) under obligation to pay private
respondent the sum of P182,200.00 corresponding to the value of the supposed necessary and
useful improvements, as well as the pronouncement therein regarding private respondent's
right of retention, are hereby SET ASIDE. With costs against petitioner."
ISSUE/S: Whether or not Adelfo Maceda ia a possessor in good faith.
HELD:
According to Article 546 of the Civil Code Maceda was not a possessor in good faith, i.e., one
who possesses in concept of an owner; hence, he had no right to retain possession of the
leased premises pending reimbursement of his improvements thereon. No lessee can claim
to be a possessor in good faith.
The promise of the now deceased spouses Arturo Victoria and Maxima Monserrat, to
reimburse Maceda for his improvements was limited only to the initial remodeling job which
cost P40, 000, pictures of which he sent to the Victorias and which they approved and
premised to reimburse. No similar promise to pay may be implied with regard to the
additional improvements which he made without their approval and which were evidently
intended to improve them out of their property.

Del Campo v. Abesia


G.R. No. L-49219
April 15, 1988
Gancayco
FACTS: This case involves a parcel of land with an area of only about 45 square meters, situated.
An action for partition was filed by plaintiffs in the CFI of Cebu. Plaintiffs and defendants are coowners pro indiviso of this lot in the proportion of 2/3 and 1/3 share each, respectively. The
houses of plaintiffs and defendants were surveyed and shown on the sketch plan. The house of
defendants occupied the portion with an area of 5 square meters of Lot of plaintiffs. The parties
manifested their conformity to the report and asked the trial court to finally settle and
adjudicate who among the parties should take possession of the 5 square meters of the land in
question.
The trial court held that the rights of a builder in good faith under Article 448 of the New Civil
Code does not apply to a case where one co-owner has built, planted or sown on the land
owned in common. Also, the defendants are ordered at their expense to remove and demolish
part of their house which has encroached an area of five (5) square meters from the lot of the
plaintiffs.
ISSUE: WoN Art. 448 would apply in properties owned in common
HELD: NO. Article 448 of the Civil Code cannot apply where a co-owner builds, plants or sows
on the land owned in common for then he did not build, plant or sow upon land that exclusively
belongs to another but of which he is a co-owner. The co-owner is not a third person under the
circumstances, and the situation is governed by the rules of co-ownership.
However, when, as in this case, the co-ownership is terminated by the partition and it appears
that the house of defendants overlaps or occupies a portion of 5 square meters of the land
pertaining to plaintiffs which the defendants obviously built in good faith, then the provisions of
Article 448 of the new Civil Code should apply.
Applying the afore-said provision of the Civil Code, the plaintiffs have the right to appropriate
said portion of the house of defendants upon payment of indemnity to defendants as provided
for in Article 546 of the Civil Code. Otherwise, the plaintiffs may oblige the defendants to pay
the price of the land occupied by their house. However, if the price asked for is considerably
much more than the value of the portion of the house of defendants built thereon, then the
latter cannot be obliged to buy the land. The defendants shall then pay the reasonable rent to
the plaintiffs upon such terms and conditions that they may agree. In case of disagreement, the
trial court shall fix the terms thereof. Of course, defendants may demolish or remove the said
portion of their house, at their own expense, if they so decide.

ARANGOTE vs. MAGLUNOB


G.R. No. 178906
February 18, 2009
FACTS:
Esperanza Maglunob-Dalisan executed an affidavit dated June 9, 1986 whereby she
renounced, relinquished, waived and quitclaimed all her rights, share, interest and
participation whatsoever over a parcel of land, in favor of the spouses Ray Mars Arangote and
Elvira T. Arangote, together with their heirs, successors and assigns including the improvements
thereon. In 1989, petitioner spouses Arangote constructed a house on the subject property,
and an OCT was issued in their favor on March 1993. However, respondent spouses Maglunob
and Romeo Salido built a hollow wall behind and in front of petitioners house preventing the
entrance to its main door. The petitioners filed a complaint in the MTC and alleged that
Esperanza inherited the subject property from her uncle Victorino Sorrosa by virtue of a
notarized Partition Agreement dated 29 April 1985, executed by the latters heirs. Thereafter,
Esperanza declared the subject property in her name for real property tax purposes, as
evidenced by Tax Declaration No. 16218 (1985). Respondents averred that they co-owned the
subject property with Esperanza, by virtue of succession. Esperanza together with her siblings
inherited the property, from their father Martin Maglunob (Martin I). When Tomas and
Inocencia passed away, their shares passed on by inheritance to respondents Martin II and
Romeo, respectively. Hence, the subject property was co-owned by Esperanza, respondent
Martin II (together with his wife Lourdes), and respondent Romeo, each holding a one-third
pro-indiviso share therein. Thus, Esperanza could not validly waive her rights and interest over
the entire subject property in favor of the petitioner.
Issue:
1. Whether or not petitioners are possessor in good faith.
2. Whether or not the petitioners are builders in good faith.

Held:
No. Article 526 of the Civil Code provides that a person is deemed a possessor in good
faith if he is not aware that there exists in his title or mode of acquisition any flaw which
invalidates it. Possession in good faith ceases from the moment defects in the title are made
known to the possessor. The moment the respondents filed a complaint that the subject
property was co-owned by them with Esperanza, good faith of the petitioners had ceased. The
petitioners are not a builder in good faith based on Article 448 of the Civil Code. In this case, the
subject property waived and quitclaimed by Esperanza to the petitioner and her husband in the
Affidavit was only covered by a tax declaration in the name of Esperanza. Petitioner did not
even bother to look into the origin of the subject property and to probe into the right of
Esperanza to relinquish the same. Thus, when petitioner and her husband built a house
thereon in 1989 they cannot be considered to have acted in good faith as they were fully aware

that when Esperanza executed an Affidavit relinquishing in their favor the subject property the
only proof of Esperanzas ownership over the same was a mere tax declaration.

Title: Meneses VS Court of Appeals


G.R. No. : 82220
Date: July 14, 1995
Ponente: Quiason, J.
FACTS: On March 1, 1977, Darum, then the District Land Officer of Los Baos, Laguna, issued to
Pablito Meneses 2 Free Patent and 2 OCT covering lots located in Los Baos, Laguna.
Pablito acquired said property from Bautista through a Deed of Waiver and Transfer of Rights
executed in 1975 in consideration of Bautistas love and affection for and some monetary
obligations in favor of Meneses. After the execution of said document, Meneses took
possession of the land, introduced improvements thereon, declared the land as his own for tax
purposes and paid the corresponding realty taxes. In turn, Bautista acquired the land from his
aunt. He had been occupying the land since 1956.
On the other hand, the Quisumbing family traces ownership of their land as far back as 1919
when their matriarch was issued an OCT covering a lot, with the Laguna de Bay as its
northwestern boundary. The same parcel of land was registered on 1973 under a TCT in the
names of her heirs, all surnamed Quisumbing.
The Quisumbings applied for registration and confirmation of title over an additional area
which had gradually accrued to their property by the natural action of the waters of Laguna de
Bay. The CFI of Bian confirmed the Quisumbings title thereto.
In 1979, the Quisumbings filed a case before the CFI of Calamba against Lorenzo and Pablito
Meneses, Darum and Almendral for nullification of the free patents and titles issued to Pablito
Meneses. They alleged that Lorenzo Menesis, then the Mayor of Los Baos, using his brother
Pablito as a tool and dummy, illegally occupied their private accretion land and
confederating with District Land Officer Darum and Land Inspector Almendral, obtained free
patents and OCTs to the land.
In 1984, the trial court rendered the decision finding that the lands registered by the Meneses
brothers are accretion lands to which the Quisumbings have a valid right as owners of the
riparian land to which nature had gradually deposited the disputed lots. (The lots occupied by
Meneses, as found by the court, are to be accretion lands forming parts of the bigger accretion
land owned by the Quisumbings. )
Meanwhile, the Meneses brothers and Darum appealed the to the CA, which affirmed in toto
the lower courts decision.The defendants-appellants filed two MRs of the CA decision but it
was denied, hence this petition for review on certiorari.

ISSUE/S: WON The lands in question were not accretion lands but lands of the public domain
HELD: WHEREFORE, the petition is DENIED. The Decision CA is AFFIRMED

While the lots occupied by Villamor and Lanuzo may not be the very same lots petitioners are
claiming here, the two cases refer to the same accretion lands northwest of the original land
owned by the Quisumbings.
The submersion in water of a portion of the land in question is due to the rains falling directly
on or flowing into Laguna de Bay from different sources. Since the inundation of a portion of
the land is not due to flux and reflux of tides it cannot be considered a foreshore land. The
land sought to be registered not being part of the bed or basin of Laguna de Bay, nor a
foreshore land as claimed by the Director of Lands, it is not a public land and therefore capable
of registration as private property provided that the applicant proves that he has a registerable
title.
Additionally, the provision of the law on waters will govern in determining the natural bed or
basin of the lake. And accordingly, to Art. 84 of the Law of Waters of August 3, 1866:
Accretions deposited gradually upon land contiguous to creeks, streams, rivers and lakes by
accessions or sediments from the waters thereof, belong to the owners of such lands.
As pointed out by the lower court, no act of appropriation is necessary in order to acquire
ownership of the alluvial formation as the law does not require the same.
VDA. DE NAZARENO VS. COURT OF APPEALS
G.R. NO. 98045.
JUNE 26, 1996
ROMERO, J.:
FACTS:
A parcel of land situated in Puntod, Cagayan de Oro City is said to have been formed as a result
of sawdust dumped into the dried-up Balacanas Creek and along the banks of the Cagayan
river. In 1979, Jose Salasalan and Leo Rabaya leased the subject lots on which their houses
stood, from one Antonio Nazareno. In 1982, Salasalan and Rabaya stopped paying rentals. As a
result, , Nazareno et. al. filed a case for ejectment with the MTC. A decision was rendered
against Salasalan and Rabaya, which decision was later affirmed by the RTC. Consequently,
Salasalan and Rabaya filed a case for annulment of judgment, petitionfor certiorari with prayer
for restraining order and/or writ of preliminary injunction which was all dismissed. The
decision of the lower court was finally enforced with the Salasalan and Rabaya being ejected
from portions of the subject lots they occupied. Before he died, Nazareno caused the approval
by the Bureau of Lands of the survey plan with a view to perfecting his title over the accretion
area being claimed by him. Before the approved survey plan could be released to the applicant,
however, it was protested by Salasalan and Rabaya. The Bureau of Lands then rendered a
decision ordering the amendment of the survey plan in the name of Nazareno by segregating
therefrom the areas occupied by the Salasalan and Rabaya. Nazareno filed a motion for
reconsideration which was denied. Later on, Desamparado Vda. de Nazareno and Leticia Tapia
Nazareno, filed a case with the RTC for the annulment of the verification, report and
recommendation, decision and order of the Bureau of Lands. The RTC dismissed said case
which was later on appoved by the CA. Hence, this petition.

ISSUE:
Whether or not the subject land is private land being an accretion to Antonio Nazarenos titled
property.
HELD:
No. Accretion, as a mode of acquiring property under Art. 457 of the Civil Code, requires the
concurrence of three requisites: (1) that the deposition of soil or sediment be gradual and
imperceptible; (2) that it be the result of the action of the waters of the river (or sea); and (3)
that the land where accretion takes place is adjacent to the banks or rivers (or the sea
coast). These are called the rules on alluvion which if present in a case, give to the owners of
lands adjoining the banks of rivers or streams any accretion gradually received from the effects
of the current of waters. The application of the rules on alluvion cannot be made in the present
case as the first and second requirements of the rules were not met. By their own admission,
the accretion was formed by the dumping of boulders, soil and other filling materials on
portions of the Balacanas Creek and the Cagayan River. Therefore, it cannot be claimed that
that the accumulation of such boulders, soil and other filling materials was gradual and
imperceptible, resulting from the action of the waters of the current of the Balacanas Ceek and
the Cagayan River. Thus, the Nazarenos cannot claim the rights of a riparian owner.
Title: Jagualing vs CA
G.R. No.: 94283
Date: March 4, 1991
Ponente: Gancayco, J.
FACTS: The parties to this case dispute the ownership of a certain parcel of land in Misamis
Oriental forming part of an island in a non-navigable river. The appellant (Janita Eduave, the
private respondent) claims that she inherited the land from his father, Felomino Factura,
together with his co-heirs and acquired sole ownership of the property by virtue of a Deed of
Extra Judicial Partition with sale. The land is declared for tax purposes under Tax Declaration
No. 26137.Since the death of her father, Eduave had been in possession of the property
although the tax declaration remains in the name of the deceased father.
The land was eroded due to typhoon Ineng, destroying the bigger portion and the
improvements leaving only a coconut tree. Due to the movement of the river deposits on the
land that was not eroded increased the are to almost half a hectare and the appellant started
to plant bananas. The defendant-appellees then asked her permission to plant corn and
bananas provided that they prevent squatters to come to the area.
The land was the subject of a reconveyance case between the appellant Janita Eduave vs. Heirs
of Antonio Factura. The heirs of Antonio Factura, who are presently the defendants-appellees in
this case, had ceded a portion of the land to Eduave pursuant to the decision of the Court of
First Instance. The defendants-appellees (petitioners herein) denied the claim of ownership of
the appellant, and asserted that they are the real owners of the land. During the typhoon Ineng
in 1964 the river control was washed away causing the formation of an island, which is now the
land in litigation. The defendants started occupying the land in 1969.

On 17 July 1987 the trial court dismissed the complaint for failure of private respondents as
plaintiffs therein to establish by preponderance of evidence their claim of ownership over the
land in litigation. The court found that the island is a delta forming part of the river bed which
the government may use to reroute, redirect or control the course of the Tagoloan River.
Accordingly, it held that it was outside the commerce of man and part of the public domain,
citing Article 420 of the Civil Code. As such it cannot be registered under the land registration
law or be acquired by prescription. The trial court, however, recognized the validity of
petitioners' possession and gave them preferential rights to use and enjoy the property. The
trial court added that should the State allow the island to be the subject of private ownership,
the petitioners have rights better than that of private respondents.
On appeal to the Court of Appeals, respondent court found that the island was formed by the
branching off of the Tagoloan River and subsequent thereto the accumulation of alluvial
deposits. Basing its ruling on Articles 463 and 465 of the Civil Code the Court of Appeals
reversed the decision of the trial court, declared private respondents as the lawful and true
owners of the land subject of this case and ordered petitioners to vacate the premises and
deliver possession of the land to private respondents.
ISSUE/S: Who between the one who has actual possession of an island that forms in a nonnavigable and non-floatable river and the owner of the land along the margin nearest the island
has the better right thereto
HELD: The parcel of land in question is part of an island that formed in a non-navigable and
non-floatable river; from a small mass of eroded or segregated outcrop of land, it increased to
its present size due to the gradual and successive accumulation of alluvial deposits. In this
regard the Court of Appeals also did not err in applying Article 465 of the Civil Code. Under this
provision, the island belongs to the owner of the land along the nearer margin as sole owner
thereof; or more accurately, because the island is longer than the property of private
respondents, they are deemed ipso jure to be the owners of that portion which corresponds to
the length of their property along the margin of the river.
It is well-settled that lands formed by accretion belong to the riparian owner. This preferential
right is, under Article 465, also granted the owners of the land located in the margin nearest the
formed island for the reason that they are in the best position to cultivate and attend to the
exploitation of the same.
Petitioners may therefore, acquire said property by adverse possession for the required
plumber of years under the doctrine of acquisitive prescription. Their possession cannot be
considered in good faith, however, because they are presumed to have notice of the status of
private respondents as riparian owners who have the preferential right to the island as
recognized and accorded by law; they may claim ignorance of the law, specifically Article 465 of
the Civil Code, but such is not, under Articles 3 and 526 of the same code, an adequate and
valid defense to support their claim of good faith. Hence, not qualifying as possessors in good
faith, they may acquire ownership over the island only through uninterrupted adverse
possession for a period of thirty years. By their own admission, petitioners have been in
possession of the property for only about fifteen years. Thus, by this token and under the
theory adopted by petitioners, the island cannot be adjudicated in their favor.

AGUSTIN VS INTERMEDIATE APPELLATE COURT


GR. Nos. L-6607-76
July 5, 1990
Grino Aquino, J.:
FACTS:
Private respondents, Maria Melad and Pablo Binuyag are among those who are
occupying the western bank of the Cagayan River while on the eastern bank is owned by
petitioner Eulogio Agustin. From 1919 to 1968, the Cagayan river has eroded the lands on the
eastern bank including Agustins Lot depositing alluvium on the land possessed by Pablo
Binuyag. In 1968, after a typhoon which caused a big flood, the Cagayan River changed its
course and returned it to its 1919 bed and it cut through the lands of respondents whose lands
were transferred on the eastern side. To cultivate the lands they had to cross the river. When
they were cultivating said lands, (they were planting corn) Agustin accompanied by the mayor
and some policemen claimed the land and drove them away. So Melad and Binuyag filed
separate complaints for recovery of their lots and its accretions. The Trial Court held ordered
Agustin et. al to vacate the lands and return them to respondents. On appeal, the IAC affirmed
in toto the judgment thus the case at bar.
ISSUE:
Whether or not private respondents own the accretion and such ownership is not affected by
the sudden and abrupt change in the course of the Cagayan River when it reverted to its old
bed
HELD: YES
Art. 457 states that the owner of the lands adjoining river banks own the accretion
which they gradually receive from the effects of the currents of the waters. Accretion benefits a
riparian owner provided that these elements are present:
1) deposit be gradual and imperceptible
2) it resulted from the effects of the current of the water and
3) the land is adjacent to the river bank.
When the River moved from 1919 to 1968, there was alluvium deposited and it was gradual
and imperceptible.
Accretion benefits the riparian owner because these lands are exposed to floods and
other damage due to the destructive force of the waters, and if by virtue of law they are subject
to encumbrances and various kinds of easements, it is only just that such risks or dangers
should in some way be compensated by the right of accretion. Also, respondents ownership
over said lots was not removed when due to the sudden and abrupt change in the course of the
river; their accretions were transferred to the other side. Art. 459 states when the current of a
river:

xxx
segregates from an estate on its bank a known portion of land and transfers it to
another estate, the owner of segregated portion retains ownership provided he removes the
same w/in 2 years.
And Art. 463 states that whenever the current of a river divides itself into branches, leaving a
piece of land or part thereof isolated, the owner of the land retains ownership. He also retains
it if a portion of land is separated from the estate by the current.

Tuason vs CA
GR No. L-48297; GR No. L-48265
Date: January 7 1987
Ponente: Paras, J
Facts:
The estate of Juan Estevez had accumulation of alluvial deposits which he took
possession of by constructing a residential house but which was subsequently destroyed by a
typhoon. Diogenes Tuazon alleges ownership over a portion of that parcel of land of the estate
because he purchased the house of Patricio Estevez which was situated on a portion of the
disputed lot. He applied w/ Director of Lands to purchase the second parcel of land under
mistaken belief that it was disposable public land, w/c estate of Estevez protested against, as
they claimed ownership over the land by right of accretion. Case was brought to RTC and was
ruled in favor of plaintiff. CA affirmed.
Issue: WON Article 457 of the New Civil Code relating to the right of a riparian owner of real
properties over the additional estate formed therein by accretion applies in the case at bar
Held:
Based on conclusive evidence, Lot 109 was declared to have been formed by the alluvial
accumulation of deposit through the action of the current of the river. Worth mentioning also is
the fact that the same trial judge who made the factual finding that Kapantawan River did not
exist in the area, granted a motion for new trial to receive newly discovered evidence.
Therefore Article 457 of the Civil Code of the Philippines applies. Consequently, the Director of
Lands has no jurisdiction over it and any conveyance made by him of any private land is null and
void.
Felicidad Javier vs. Regino Veridiano and Rosete
G.R. No. L-48050
October 10, 1994
Bellosillo, J.
FACTS:

On 25th of January 1963, Javier filed a Miscellaneous Sales Application for Lot No. 1641, Ts 308
of Olongapo Townsite Subdivision in Olongapo City. On December 1970, Javier filed Civil Case
no. 926 (CC 926) for forcible entry against Ben Babol for entering a portion of the southwestern
part of Lot 1641, Ts 308. The case was dismissed since the court considered the portion outside
Lot 1461. The case became final and executory on April 1973. Subsequently, Javier was granted
Miscellaneous Sales Patent and an Original Certificate of Title was issued in her favor.
Meanwhile, Babol who was the defendant in CC 926 had sold the portion he was occupying to
Rosete. 4 years after the finality of CC 926, Javier instituted a Civil Case No. 2203-0 (CC 2203-0)
for quieting of title and recovery of possession. Rosete moved to dismiss on the ground of res
judicata. The CFI of Zambales dismissed the case.
ISSUE:
Whether or not res judicata is applicable in the case?
HELD:
No. The following are the requisites of res judicata: a) there is final judgment or order; b) the
court have jurisdiction over the subject matter; c) former judgment is a judgment on merits;
and d) identity of parties, of subject matter, and of causes of action. The first three are present.
There is identity of parties in the case. What is required is not absolute but substantial identity
of parties. In the case, Rosete is a successor in interest of Babol by title. Nevertheless, there is
no identity of cause of action. CC 926 is a complaint of forcible entry or accion interdictal where
the issue is physical or material possession of real property. In this case, Javier merely claimed a
better right or prior possession over the land without asserting title. CC 2203-0 is an action to
recover a parcel of land or accion reivindicatori. In this case, Javier expressly alleged ownership
(by virtue of the Original Certificate of Title issued) and specifically prayed that she be declared
the rightful owner and be given possession of the disputed portion. A judgement in forcible
entry or detainer case disposes of no other issue than possession and declares only who has the
right of possession, but by no means constitutes a bar to an action for determination of who
has the right or title of ownership.
Title: VDA. DE ROXAS v. OUR LADYS FOUNDATION, INC
GR No.: 182378
Date: March 6, 2013
Ponente: C.J. Sereno
FACTS:
Salve Dealca Latosa was the owner of a residential land located at Our Lady's Village,
Bibincahan, Sorsogon, Sorsogon. Latosa filed a case against Atty. Henry Amado Roxas for the
recovery of ownership of a portion of her residential land. According to her, Atty. Roxas,
represented by petitioner herein, encroached on a quarter of her property by arbitrarily
extending his concrete fence beyond the correct limits.
In his Answer, Roxas imputed the blame to respondent Our Lady's Foundation, Inc.
(OLFI), claiming that he only occupied the adjoining portion in order to get the equivalent area
of what he had lost when OLFI trimmed his property for the subdivision road.

The RTC held that Latosa had established her claim of encroachment and found that
Roxas occupied a total of 112 square meters of Latosa's lots, and that, in turn, OLFI trimmed his
property by 92 square meters. The court ordered Roxas to return the portion encroached by
him and to demolish the structure he constructed. The court also ordered OLFI to reimburse
Roxas for the value of the 92 sq. meters that was trimmed from his property, the value of which
should be reckoned from the time of payment to the 3rd party defendant.
Respondent OLFI then filed a Motion to Quash the Sheriff's Bill, opposing the valuation
of the subject property and insisted that it should reimburse Roxas only at the rate of P40 per
square meter, the same rate that Roxas paid when the latter first purchased the property. The
trial court denied the motion and approved an Amended Sheriff's Bill with the valuation of
P1,800 per square meter.
Respondent OLFI still refusing to pay P1,800 per square meter to Roxas, filed a Rule 65
Petition before the CA asserting that they should only reimburse the purchase price that Roxas
had originally paid, which was P40 per square meter for the 92-square-meter property.
The CA then ruled in favor of OLFI and rendered judgment that the amount should only
be P40 per sq. meter, which is the amount reckoned from the original purchase price.
Hence, this petition.
ISSUE:
W/N the value for reimbursement should be based on the original purchase price or the current
fair market value
HELD:
The court held that the value for reimbursement should be based on the current fair
market value of the property. Under Article 448 pertaining to encroachments in good faith, as
well as Article 450 referring to encroachments in bad faith, the owner of the land encroached
upon has the option to require respondent builder to pay the price of the land.
The court has ruled in several cases that in the event that the seller elects to sell the lot,
the price must be fixed at the prevailing market value at the time of payment. The present or
current fair value of the land is to be reckoned at the time that the landowner elected the
choice, and not at the time that the property was purchased.
The CA incorrectly pegged the reimbursable amount at the old market value of the
subject property P40 per square meter as reflected in the Deed of Absolute Sale between
the parties. On the other hand, the RTC properly considered the value of the lot at P1,800 per
square meter, the current fair price as determined in the Amended Sheriff's Bill. Thus, we
reverse the ruling of the CA and reinstate the order of the RTC directing OLFI to reimburse
petitioner at P1,800 per square meter.
The value of the 92-square-meter property for which respondent should reimburse
petitioner, as determined by the RTC is hereby reinstated at P1,800 per square meter.

TITLE: BALLATAN vs. CA


G.R. No. 125683
DATE: March 2, 1999
PONENTE: PUNO, J.

FACTS: The parties herein are owners of adjacent lots. Lot 24 is registered in the name of
petitioners Ballatan. Lots 25 & 26 are registered in the name of respondent Go Sr. His son,
Winston Go constructed a house on Lot 25. Adjacent to Lot 26 is Lot 27, 4 registered in the
name of respondent Li Ching Yao.
Petitioner Ballatan constructed her house on Lot 24. During the construction, she noticed that
the concrete fence and side pathway of respondents house encroached her property. Ballatan
informed Go of this discrepancy and his encroachment on her property. Surveys were made
and it was found out that the lot area of petitioner Ballatan was less by a few meters and that
of respondent Li Ching Yao, which was three lots away, increased by two 2 meters.
Petitioner Ballatan made a written demand on respondents Go to remove and dismantle their
improvements on Lot 24. Respondents Go refused. Hence, Ballatan filed for recovery of
possession of real property. TC decided in favor of petitioners. It ordered the Go's to vacate the
subject portion, demolish their improvements. CA modified: ordered Go to pay Ballatan, and
respondent Li Ching Yao to pay Go and the value to be fixed at the time of the taking.
ISSUE: WON the value should be fixed at the time of the taking.
HELD: NO. All the parties are presumed to have acted in good faith. Their rights must,
therefore, be determined in accordance with the appropriate provisions of the Civil Code on
property specifically Art. 448. Petitioners, as owners of Lot 24, may choose to purchase the
improvement made by respondents Go on their land, or sell to respondents Go the subject
portion. If buying the improvement is impractical as it may render the Go's house useless, then
petitioners may sell to respondents Go that portion of Lot 24 on which their improvement
stands. If the Go's are unwilling or unable to buy the lot, then they must vacate the land and,
until they vacate, they must pay rent to petitioners. Petitioners, however, cannot compel
respondents Go to buy the land if its value is considerably more than the portion of their house
constructed thereon. If the value of the land is much more than the Go's improvement, then
respondents Go must pay reasonable rent. If they do not agree on the terms of the lease, then
they may go to court to fix the same.
In the event that petitioners elect to sell to respondents Go the subject portion of their lot, the
price must be fixed at the prevailing market value at the time of payment.
Geminiano v. Court of Appeals
G.R. No. 120303
July 24, 1996
DAVIDE, JR., J.:
FACTS: It appears that subject lot was originally owned by the petitioners' mother, Paulina
Amado vda. de Geminiano. On a 12-square-meter portion of that lot stood the petitioners'
unfinished bungalow, which the petitioners sold to the private respondents, with an alleged
promise to sell to the latter that portion of the lot occupied by the house. Subsequently, the

petitioners' mother executed a contract of lease over a 126 square-meter portion of the lot,
including that portion on which the house stood, in favor of the private respondents for
P40.00per month for a period of 7 years.
The private respondents then introduced additional improvements and registered the house in
their names. After the expiration of the lease contract, however, the petitioners' mother
refused to accept the monthly rentals.
It turned out that the lot in question was the subject of a suit, which resulted in its acquisition
by one Maria Lee in 1972. Lee sold the lot to Lily Salcedo, who in turn sold it to the spouses
Dionisio. Spouses Dionisio executed a Deed of Quitclaim over the said property in favor of the
petitioners.
The petitioners sent a letter addressed to private respondent Mary Nicolas demanding that she
vacate the premises and pay the rentals in arrears within twenty days from notice.
Upon failure of the private respondents to heed the demand, the petitioners filed a complaint
for unlawful detainer and damages.
ISSUE: Whether or not Art. 448 is applicable to this case.
HELD: NO. The private respondents claim they are builders in good faith, hence, Article 448 of
the Civil Code should apply. They rely on the lack of title of the petitioners' mother at the time
of the execution of the contract of lease, as well as the alleged assurance made by the
petitioners that the lot on which the house stood would be sold to them. But being mere
lessees, the private respondents knew that their occupation of the premises would continue
only for the life of the lease. Plainly, they cannot be considered as possessors nor builders in
good faith.
Article 448 of the Civil Code, in relation to Article 546 of the same Code, which allows full
reimbursement of useful improvements and retention of the premises until reimbursement is
made, applies only to a possessor in good faith, i.e., one who builds on land with the belief that
he is the owner thereof. It does not apply where one's only interest is that of a lessee under a
rental contract; otherwise, it would always be in the power of the tenant to "improve" his
landlord out of his property.
And even if the petitioners indeed promised to sell, it would not make the private respondents
possessors or builders in good faith so as to be covered by the provisions of Article 448 of the
Civil Code. The latter cannot raise the mere expectancy of ownership of the aforementioned lot
because the alleged promise to sell was not fulfilled nor its existence even proven.
RACAZA vs. SUSANA REALTY, INC.
G.R. No. L-20330 December 22, 1966
J. REGALA
FACTS:
Petitioner is the lessee of a portion of a piece of land located at San Juan St., Pasay City,
and owned by respondent corporation. He started renting this portion of the lot in 1952 when
his wife, Evarista P. Racaza, bought an unfinished house that had been built on it. On assurance
of respondent that petitioner's family could stay on the land by paying a monthly rent of P15,
petitioner finished the construction of the house and he and his family lived in it. On December
16, 1955, however, petitioner was asked to vacate the land because respondent needed it. The

demand was followed by the filing on February 10, 1956 of a complaint for ejectment in the
Municipal Court of Pasay City. Petitioner and his family remained in the premises as the case
was dismissed for failure of respondent to proceed to trial.
On December 17, 1957, petitioner received another letter from respondent demanding
anew the surrender of the premises. On February 19, 1958, another ejectment suit was filed
against him, the complaint alleging that respondent needed the lot "for the purpose of
constructing improvements thereon and for other uses but that despite repeated demands
petitioner refused to leave the premises.
In his answer, petitioner denied that the lease was on a month-to-month basis and
claimed that his understanding with respondent was that he would be allowed to stay on the
premises as long as he paid a monthly rent of P15. As counterclaim, petitioner demanded the
payment of P12,000 which he said he had spent to finish the construction of his house.
ISSUE:
Whether or not petitioner may ask for reimbursement for the improvements of the
leased property
HELD:
There IS NO merit in petitioner's last point that he should have been allowed full
reimbursement for what he had spent by applying to this case article 448 of the Civil Code. It is
now settled that article 448, in relation to article 546, applies only to possessors in good faith
and since lessees, like petitioner, are not possessors in good faith, because they know that their
occupation of the premises continues only during the life of the lease, they cannot recover the
value of their improvements from the lessor, much less retain the premises until they are
reimbursed. Their rights are governed by article 1678 which allows reimbursement of lessees
up to one-half of the value of their useful improvements. (Lopez, Inc. vs. Philippine & Eastern
Trading Co., 98 Phil. 348 [1956]) The Court of Appeals correctly applied article 1678 to this case.
Accordingly, upon the expiration of the lease in this case, petitioner became a
deforciant unlawfully withholding possession of the property. There was no need for a demand
to be served on him, except to negate any inference that respondent, as lessor, had agreed to
an extension of the term of the lease under article 1687 of the Civil Code
SANTOS VS. COURT OF APPEALS
G.R. NO. 100963
APRIL 6, 1993
CAMPOS, JR., J p:

FACTS:

Margarita Tuason leased a parcel of land in favor of Fortunato Santos, predecessor-ininterest of herein petitioners, for a period of three (3) years from July 1951 to July 1954. As a
result, Fortunato Santos took possession of the land and erected a residential house and a
barbershop thereon.
Fortunato Santos continued to occupy the premises even after the expiration of the
lease contract on July 1954. In the meantime, the successor-in-interest of the deceased lessor,
Asuncion Tuason, conveyed the property to private respondent Lolita Deopante Vda. De
Saavedra.
Upon the death of Fortunato Santos, petitioners continued their possession of the
premises but failed to pay the monthly rentals thus prompting the private respondent to file an
ejectment case with the Pasig Metropolitan Trial Court (MTC).
The MTC ruled in favor of the private respondent and the decision was affirmed by the
Regional Trial Court (RTC). However, the RTC added that herein petitioners be reimbursed for
the improvements upon the land. The Court of Appeals modified the decision of the RTC,
stating that the reimbursements to be made by the private respondent of the improvements
introduced to the leased premises be eliminated.
ISSUE:
Whether or not, herein petitioners have the right to be reimbursed for the
improvements made by Fortunato Santos.
HELD:
The Supreme Court held that in the case at bar, although the lessee is a lawful possessor
because his entry into the premises is by virtue of the lease contract, he is not a possessor in
good faith as his possession is with the consent of the owner of the property whose title he
recognizes by the lease contract. Hence, he cannot claim the right of reimbursement or
retention to which a possessor in good faith is entitled. His rights are governed by Article 1678
of the Civil Code, which allows reimbursement of lessees up to one-half of the value of their
useful improvements.
Under Article 1678, it is the lessor who has the option to pay for one-half of the value of
the improvements, which the lessee has made in good faith, which are suitable for the use for
which the lease is intended and which have not altered the form and substance of the land. The
lessee cannot compel the lessor to appropriate and reimburse. His right is to remove the
improvement even though it may suffer damage thereby.

Sps. Verona and Ricardo Kilario vs CA and Silverio Pada


Gr No. 134329
January 19, 2000
Ponente: De Leon, Jr. J.:
Facts:
Jacinto Pada had 6 children, namely, Marciano, Ananias, Amador, Higino, Valentina and
Ruperta. He died intestate. His estate included a parcel of land of residential and coconut land
located at Poblacion, Matalom, Leyte, denominated as Cadastral Lot No. 5581 with an area of
1,301.92 square meters. It is the northern portion of Cadastral Lot No. 5581 which is the subject
of the instant controversy.
During the lifetime of Jacinto Pada, his half-brother, Feliciano Pada, obtained permission from
him to build a house on the northern portion of Cadastral Lot No. 5581. When Feliciano died,
his son, Pastor, continued living in the house together with his eight children. Petitioner Verona
Pada-Kilario, one of Pastor's children, has been living in that house since 1960.
Sometime in May, 1951, the heirs of Jacinto Pada entered into an extra-judicial partition of his
estate. For this purpose, they executed a private document which they, however, never
registered in the Office of the Registrar of Deeds of Leyte.
On June 14, 1978, Juanita Pada sold to Engr. Ernesto Paderes, the right of his father, Ananias, as
co-owner of Cadastral Lot No. 5881.
On November 17, 1993, it was the turn of Maria Pada to sell the co-ownership right of his
father, Marciano. Private respondent, who is the first cousin of Maria, was the buyer.
Thereafter, private respondent demanded that petitioner spouses vacate the northern portion
of Cadastral Lot No. 5581 so his family can utilize the said area.
On June 26, 1995, private respondent filed in the Municipal Circuit Trial Court of Matalom,
Leyte, a complaint for ejectment with prayer for damages against petitioner spouses.
On July 24, 1995, the heirs of Amador Pada, executed a Deed of Donation transferring to
petitioner Verona Pada-Kilario, their respective shares as co-owners of Cadastral Lot No. 5581.
The Municipal Circuit Trial Court rendered judgment in favor of petitioner spouses. Private
respondent appealed to the Regional Trial Court. On November 6, 1997, it rendered a judgment
of reversal. Petitioners filed in the Court of Appeals a petition for review of the foregoing
decision of the Regional Trial Court.
On May 20, 1998, respondent Court of Appeals rendered judgment dismissing said petition.
Hence, the petition
Issues:
1. Whether or not the petitioners, as co-owners, can be ejected from the premises
2. Whether or not maria pada has the right to sold the undivided share in the property
3. Whether or not the petitioners is builders in good faith
Hold: No merit to the instant petition.
We hold that the extrajudicial partition of the estate of Jacinto Pada among his heirs
made in 1951 is valid. No law requires partition among heirs to be in writing and be registered
in order to be valid. The requirement in Sec. 1, Rule 74 of the Revised Rules of Court that a
partition be put in a public document and registered, has for its purpose the protection of
creditors and the heirs themselves against tardy claims. The object of registration is to serve as

constructive notice to others. It follows then that the intrinsic validity of partition not executed
with the prescribed formalities is not undermined when no creditors are involved. Without
creditors to take into consideration, it is competent for the heirs of an estate to enter into an
agreement for distribution thereof in a manner and upon a plan different from those provided
by the rules from which, in the first place, nothing can be inferred that a writing or other
formality is essential for the partition to be valid. The partition of inherited property need not
be embodied in a public document so as to be effective as regards the heirs that participated
therein. The requirement of Article 1358 of the Civil Code that acts which have for their object
the creation, transmission, modification or extinguishment of real rights over immovable
property, must appear in a public instrument, is only for convenience, non-compliance with
which does not affect the validity or enforceability of the acts of the parties as among
themselves. And neither does the Statute of Frauds under Article 1403 of the New Civil Code
apply because partition among heirs is not legally deemed a conveyance of real property,
considering that it involves not a transfer of property from one to the other but rather, a
confirmation or ratification of title or right of property that an heir is renouncing in favor of
another heir who accepts and receives the inheritance. The extrajudicial partition of Jacinto
Pada's estate being legal and effective as among his heirs Juanita and Maria Pada validly
transferred their ownership rights over the land to Engr. Paderes and private respondent.
The extrajudicial partition which the heirs of Jacinto Pada executed voluntarily and
spontaneously in 1951 has produced a legal status. When they discussed and agreed on the
division of the estate Jacinto Pada, it is presumed that they did so in furtherance of their mutual
interests. As such, their division is conclusive, unless and until it is shown that there were debts
existing against the estate which had not been paid. No showing, however, has been made of
any unpaid charges against the estate of Jacinto Pada. Thus, there is no reason why the heirs
should not be bound by their voluntary acts. In the said partition, what was allocated to
Amador Pada was not the subject property which was a parcel of residential land in Sto. Nino,
Matalom, Leyte, but rather, one-half of a parcel of coconut land in the interior of Sto. Nino St.,
Sabang, Matalom, Leyte and one-half of a parcel of rice land in Itum, Sta. Fe, Matalom, Leyte.
The donation made by his heirs to petitioners of the subject property, thus, is void for they
were not the owners thereof. At any rate it is too late in the day for the heirs of Amador Pada
to repudiate the legal effects of the 1951 extrajudicial partition as prescription and laches have
equally set in.
Petitioners are estopped from impugning the extrajudicial partition executed by the
heirs of Jacinto Pada after explicitly admitting in their Answer that they had been occupying the
subject property since 1960 without ever paying any rental as they only relied on the liberality
and tolerance of the Pada family. Their admissions are evidence of a high order and bind them
insofar as the character of their possession of the subject property is concerned.
Considering that petitioners were in possession of the subject property by sheer
tolerance of its owners, they knew that their occupation of the premises may be terminated
any time. Persons who occupy the land of another at the latter's tolerance or permission,
without any contract between them, is necessarily bound by an implied promise that they will
vacate the same upon demand, failing in which a summary action for ejectment is the proper
remedy against them. Thus, they cannot be considered possessors nor builders in good faith. It
is well-settled that both Article 448 and Article 546 of the New Civil Code which allow full

reimbursement of useful improvements and retention of the premises until reimbursement is


made, apply only to a possessor in good faith, i.e., one who builds on land with the belief that
he is the owner thereof. Verily, persons whose occupation of realty is by sheer tolerance of its
owners are not possessors in good faith. Neither did the promise of Concordia, Esperanza and
Angelito Pada that they were going to donate the premises to petitioners convert them into
builders in good faith for at the time the improvements were built on the premises, such
promise was not yet fulfilled, i.e., it was a mere expectancy of ownership that may or may not
be realized. More importantly, even as that promise was fulfilled, the donation is void for
Concordia, Esperanza and Angelito Pada was not the owners of Cadastral Lot No. 5581. As such,
petitioners cannot be said to be entitled to the value of the improvements that they built on
the said lot.
WHEREFORE, the petition for review is HEREBY DENIED.

The Heirs of Sarili vs. Lagrosa


G.R. No. 193517
January 15, 2014
Perlas Bernabe, J. :
FACTS:
On February 17, 2000, respondent, represented by his attorney-in-fact Lourdes Labios
Mojica (Lourdes), via a special power of attorney, filed a complaint against Sps. Sarili and the
Register of Deeds of Caloocan City before the RTC, alleging, among others, that he is the owner
of a certain parcel of land situated in Caloocan City and has been religiously paying the real
estate taxes since its acquisition. Respondent claimed that he is a resident of California, USA,
and that during his vacation in the Philippines, he discovered that a new certificate of title (TCT
No. 262218) to the subject property was issued by the RD in the name of Victorino married to
Isabel Amparo, by virtue of a falsified Deed of Absolute Sale dated purportedly executed by him
and his wife, Amelia U. Lagrosa. He averred that the falsification of the said deed of sale was a
result of the fraudulent, illegal, and malicious acts committed by Sps. Sarili and the RD in order
to acquire the subject property and, as such, prayed for the annulment of TCT No. 262218, and
that Sps. Sarili deliver to him the possession of the subject property, or, in the alternative, that
Sps. Sarili and the RD jointly and severally pay him the amount of P1,000,000.00, including
moral damages as well as attorneys fees.
In their answer, Sps. Sarili maintained that they are innocent purchasers for value,
having purchased the subject property from Ramon B. Rodriguez, who possessed and
presented a Special Power of Attorney to sell/dispose of the same, and, in such capacity,
executed a Deed of Absolute Sale conveying the said property in their favor. In this relation,
they denied any participation in the preparation of the deed of sale, which may have been
merely devised by the "fixer" they hired to facilitate the issuance of the title in their names.
Further, they interposed a counterclaim for moral and exemplary damages, as well as
attorneys fees, for the filing of the baseless suit.
ISSUE:
Whether or not there was a valid conveyance of the subject property to Sps. Sarili.

HELD:
It is well-settled that even if the procurement of a certificate of title was tainted with
fraud and misrepresentation, such defective title may be the source of a completely legal and
valid title in the hands of an innocent purchaser for value. Where innocent third persons,
relying on the correctness of the certificate of title thus issued, acquire rights over the property,
the court cannot disregard such rights and order the total cancellation of the certificate. The
effect of such an outright cancellation would be to impair public confidence in the certificate of
title, for everyone dealing with property registered under the Torrens system would have to
inquire in every instance whether the title has been regularly or irregularly issued. This is
contrary to the evident purpose of the law.
The general rule is that every person dealing with registered land may safely rely on the
correctness of the certificate of title issued and the law will in no way oblige him to go beyond
the certificate to determine the condition of the property. Where there is nothing in the
certificate of title to indicate any cloud or vice in the ownership of the property, or any
encumbrance thereon, the purchaser is not required to explore further than what the Torrens
Title upon its face indicates in quest for any hidden defects or inchoate right that may
subsequently defeat his right thereto.
However, a higher degree of prudence is required from one who buys from a person
who is not the registered owner, although the land object of the transaction is registered. In
such a case, the buyer is expected to examine not only the certificate of title but all factual
circumstances necessary for him to determine if there are any flaws in the title of the
transferor.35 The buyer also has the duty to ascertain the identity of the person with whom he
is dealing with and the latters legal authority to convey the property.
The strength of the buyers inquiry on the sellers capacity or legal authority to sell
depends on the proof of capacity of the seller. If the proof of capacity consists of a special
power of attorney duly notarized, mere inspection of the face of such public document already
constitutes sufficient inquiry. If no such special power of attorney is provided or there is one
but there appears to be flaws in its notarial acknowledgment, mere inspection of the document
will not do; the buyer must show that his investigation went beyond the document and into the
circumstances of its execution.
Coronel vs Intermediate Appellate Court
G.R. No. 70191
October 29, 1987
Gutierrez, JR., J.:
Facts:
Rodolfo Coronel filed a complaint for recovery of possession of a parcel of land
registered under his name. The complaint was filed against the private respondents before the
Court of First Instance of Cavite. Coronel alleged in his complaint that at the time he purchased
the parcel of land, the private respondents were already occupying a portion of the land as
tenants at will and that despite demands to vacate the premises, the respondents failed and
refused to move out of the land. In their Answer, the respondents denied that Coronel was the
owner of the whole parcel of land and alleged that the lots occupied by them form part of an

undivided share of Brigido Merlan and Jose Merlan, respondents, which they inherited from
their deceased father, one of the three heirs of Bernabela Lontoc, the original owner of lot
1950-A of the estate; that the Merlan brothers never sold their share to anybody; that
Coronels claim of ownership of the whole parcel of land is fraudulent, void, and without effect;
and that the other defendants were legitimate tenants. In their Third-Party Complaint, the
defendants charged that the third-party defendants, owners of the remaining portion of Lot No.
1950-A, defrauded them when they sold the entire parcel. Third-Party Defendants Marcelo
Novelo, Paz Anuat Daniel Anuat and Rosario Cailao, the defendants' co-owners of Lot No. 1950A, denied that they had something to do with the fraudulent acts or illegal machinations which
deprived the defendants of their share in the subject parcel of land, and that what they sold
was only their 2/3 undivided shares in said parcel.
Lower court ruled in favor of the defendants and on appeal, the lower court's decision
was affirmed with modification by the then IAC.
Issue:
WON the claim of private respondents to the land in question is barred by the statute of
limitation or by estoppel by laches?
Ruling:
No.
In dispute in the instant case is the 2/8 share of Bernabela Lontoc which is equivalent to
12,189 square meter of the 48,755 square meter lot of the Naic Estate. When Lontoc died in
1945, she was survived by three sets of heirs: 1) Bernardino Merlan; 2) Jose Merlan and Brigido
Merlan; and 3) Daniel Anuat and Paz Anuat. In 1950, Bernardino Merlan, Daniel Anuat and Paz
Anuat sold their 2/3 undivided portion of the lot to spouses Ignacio Manalo and Marcela
Nobelo. Sometime in 1970, Ignacio Manalo sold his interest in Lot 1950-A to Mariano Manalo.
Considering these facts, it is evident that the private respondents never sold their 1/3 share
over Lot No. 1950-A of the Naic Estate; and that what their co-owners sold to Ignacio Manalo
was their 2/3 share of the same lot. Moreover, private respondents Brigido Merlan and Jose
Merlan were in open, peaceful and adverse possession of their 1/3 share over the lot even after
1950 when the first sale of the lot took place. The first time they knew about Coronel's claim
over the whole lot was when they were served a copy of his complaint in 1975.
The petitioner contends that the claim of the private respondents over their 1/3
undivided portion of Lot No. 1950-A 25 years after the registration of the deed of sale in favor
of Ignacio Manalo in 1950 and more than five (5) years after the registration of the deed of sale
in favor of Mariano Manalo is barred by prescription or laches. According to him, there was
undue delay on the part of the private respondents to claim their 1/3 portion of Lot No. 1950-A
of the Naic Estate and that the action for annulment should have been brought within four (4)
years (Art. 1391, New Civil Code) counted from the date of the registration of the instrument.
The counterclaim of the private respondents which was in effect a reconveyance to

them of their 1/3 undivided share over lot No. 1950-A has not prescribed. As lawful possessors
and owners of the lot in question their cause of action falls within the settled jurisprudence that
an action to quiet title to property-in one's possession is imprescriptible, Their undisturbed
possession over a period of more than 25 years gave them a continuing right to seek the aid of
a court of equity to determine the nature of the adverse claim of a third party and the effect of
his own title. It was only at that time that, the statutory period of prescription may be said to
have commenced to run against them. In the same manner, there is no bar based on laches to
assert their right over 1/3 of the disputed property. "Laches has been defined as the failure or
neglect, for an unreasonable and unexplained length of time, to do that which by exercising due
diligence could or should have been done earlier; it is negligence or omission to assert a right
within a reasonable time, warranting a presumption that the party entitled to assert it either
has abandoned it or declined to assert it. The facts of the case show that the private
respondents have always been in peaceful possession of the 1/3 portion of the subject lot,
exercising ownership thereto for more than 25 years disrupted only in 1975 when the
petitioner tried to remove them by virtue of his torrens title covering the entire Lot 1950-A of
the Naic Estate. It was only at this point that private respondents knew about the supposed sale
of their 1/3 portion of Lot 1950-A of the Naic Estate and they immediately resisted.
Ramon B. Brito Sr. vs Severino D. Dianala
G.R. No. 171717
December 15, 2010
Peralta, J
FACTS:
The subject of this case is a parcel of land in Cadiz city, Negros Occidental originally owned by a
certain Esteban Dichimo and his wife Eufemia Dianala, both of whom are already deceased. In
1976, Margarita Dichimo, assisted by her husband Ramon Brito Sr. together with Bienvenido,
Francisco, Edito, Maria, Herminia, Leonora, Felicito and Merlinda, filed civil case no. 12887 for
recovery of possession and damages with the CFI of Negros Occidental against a certain Jose
Maria Golez. Margarita, together with Bienvenido and Francisco, alleged that they are the heirs
of a certain Vicente Dichimo and that, Vicente and Eusebio are the only heirs of Esteban and
Eufemia. In 1983, herein respondents filed an answer-in-intervention claiming that Esteban,
prior to his marriage to Eufemia, was married to a certain Francisca Dumalagan and that
Esteband and Francisca bore five children all of whom are already deceased; and that herein
respondents are the heirs of Esteban and Franciscas children. Subsequently, the parties in civil
case no. 12887 entered into a compromise agreement wherein Lot 1536-B was divided
between Jose Maria Golez and the heirs of Vicente, namely: Margarita, Bienvenido and
Francisco. It was also stated in the agreement that the heirs of Eusebio had sold their share in
the said lot to the mother of Golez. TCT 12561 was thereafter issued in the name of Margarita,
Bienvenido and Francisco. In 1999, herein petitioners filed another complaint for recovery of
possession and damages this time against herein respondents. Herein respondents likewise
filed a complaint for reconveyance and damages against petitioners and his co-heirs. Both cases
were however dismissed. On appeal, judgment was rendered in favor of the respondents,
hence this petition.

ISSUE: Whether or not the respondents complaint for reconveyance is barred by prescription
HELD:
No. The complaint for reconveyance and damages filed by the respondent against the
petitioner alleged that the latter and his co-heirs acquired the subject property by means of
fraud. Art. 1456 of the civil code provides that a person acquiring property by means of fraud
becomes, by operation of law, a trustee of an implied trust for the benefit of the real owner of
the propery. An action for reconveyance based on an implied trust prescribes in 10 years. The
reckoning point of which is the date of registration of the deed or the date of the issuances of
the certificate of title over the property. In this case, TCT no. 12561 was obtained by the
petitioner and his co-heirs in 100-, while respondents filed their complaint for reconveyance in
1999. Hence, it is clear that the 10 year prescriptive period has not yet expired.

Title: GASPAR CALACALA, BALTAZAR CALACALA, MELCHOR CALACALA, SOLOMON CALACALA,


FELICIDAD CALACALA, PETRONILA CALACALA and SALOME CALACALA vs. REPUBLIC OF THE
PHILIPPINES, represented by the Solicitor General, and SHERIFF JUAN C. MARQUEZ
G.R. No.: 154415
Date: July 28, 2005
Ponente: Garcia, J:
FACTS: Camilo Calacala and Conchita Calacala are the registered owners of a land in Rosales,
Pangasinan. To secure the provisional release of an accused in a criminal case, the spouses
offered their land as a property bond. For failure of the accused to appear, the bond was
forfeited in favor of the Government. The court issued a Writ of Execution, directing the
provincial sheriff to effect a levy and sell the same at a public auction to satisfy the bond. A
notice of levy was addressed to the Register of Deeds in Pangasinan and such was annotated on
the TCT. The public auction was held and the Republic submitted its bid at P3,500. A Sheriffs
Certificate of Sale was issued in favor of the Republic. On October 5, 1982 such sale was
registered, giving the spouses one year from such date to redeem the property. They never
exercised such right until their deaths. Soon after, the legal heirs of the spouses filed with the
RTC of Rosales, Pangasinan, a complaint for Quieting of Title and Cancellaion of Encumbrance
on against the Republic. The Republic filed a motion to dismiss on the grounds of (1) failure to
state a cause of action; and (2) prescription of the petiitioners right to redeem. The lower court
granted the motion. Hence, this petition.
ISSUE/S: Whether or not the lower courts dismissal of petitioners complaint for Quieting of
Title was proper.
HELD: Yes. Under Article 476, the remedy may only be availed of when by reason of any
instrument, record, claim, encumbrance or proceeding, which appears to valid, but is, in fact,
invalid, ineffective, voidable or unenforceable, a cloud thereby casts on the complainants title
to real property or any interest therein. Under Article 477, the party who may bring an action to
quiet title must have a legal or equitable title to, or interest in the real property which is the

subject matter of the action. Verily, for an action to quiet title to prosper, two (2) indispensable
requisites must concur, namely: (1) the plaintiff or complainant has a legal or an equitable title
to or interest in the real property subject of the action; and (2) the deed, claim, encumbrance
or proceeding claimed to be casting cloud on his title must be shown to be in fact invalid or
inoperative despite its prima facie appearance of validity or legal efficacy. The foregoing
requisites are wanting in this case. The expiration of the 1-year redemption period forecloses
the obligors right to redeem and that the sale thereby becomes absolute, the issuance
thereafter of a final deed of sale is at best a mere formality and mere confirmation of the title
that is already vested in the purchaser. In this case, petitioners are no longer holders of any
legal title over the property subject of this case and are bereft of any equitable claim, which is
the very first requisite of an action to quiet title.
Aguirre et al v CA and Balitaan
G.R. No. 122249
January 29, 2004
Austria-Martinez, J.:
Facts:
Leocadio Medrano was the owner and possessor of a parcel of residential land in
Batangas, with an area of 2,611 square meters. The parcel of land was conjugal property,
having been acquired during his first marriage with one Emiliana Narito and their union begot 4
children (petitioners). After the death of his first wife, Medrano contracted a second marriage
with Miguela Cario and their union bore 4 children (co-petitioners) as well. Upon the death of
Medrano in 1945, the surviving heirs agreed that Sixto (one of the children of the 1 st marriage)
should manage and administer the subject property.
After Sixtos death, petitioners found out that in 1953, Sixto, without the knowledge and
consent of the petitioners, executed an Affidavit of Transfer of Real Property stating therein
that he was the only heir of Leocadio Medrano. Sixto sold to Maria Bacong and Tiburcio
Balitaan 160- square meter portion and 1,695 square meter portion of the disputed land
respectively. Sometime in 1967, Maria Bacong sold her property to Rosendo Bacong.
Petitioners demanded the reconveyance of the portions sold by Sixto but Balitaan,
Maria Bacong and Rosendo Bacong refused to do so. Hence, petitioners filed against them
before the RTC. The 3 purchasers contended that they acquired their property thru a valid deed
of sale; they were innocent purchaser for value; and, petitioners cause of action was barred by
prescription.
RTC upheld the sale made by Sixto in favor of private respondents only to the extent
that Sixto is entitled to by virtue of his being a co-owner. On appeal, CA declared respondents
as absolute owners of 1,695 square meters of the subject property since they have been in
possession in the concept of an owner of the entire parcel of land sold to Balitaan by Sixto
Medrano for 17 years, in the name of Sixto; and that Tiburcio acquired ownership of the whole
property from Sixto through ordinary prescription for 10 years.
Issue:
Whether or not there was a valid sale between Sixto and 3 purchasers even without
consent of co-owners.

Held:
YES, but only as to the share of Sixto. The SC held that CA committed a reversible error
in upholding the claim of petitioners that they acquired ownership of the subject property
through ordinary acquisitive prescription. If a co-owner sells the whole property as his, the
sale will affect only his own share but not those of the other co-owners who did not consent
to the sale.

Article 493 of the Civil Code provides:


Art. 493. Each co-owner shall have the full ownership of his part and the fruits and
benefits pertaining thereto, and he may therefore alienate, assign or mortgage it, and even
substitute another person in its enjoyment, except when personal rights are involved. But the
effect of the alienation or the mortgage, with respect to the co-owners, shall be limited to the
portion which may be allotted to him in the division upon the termination of the co-ownership.
It clearly provides that the sale or other disposition affects only the sellers share pro
indiviso, and the transferee gets only what corresponds to his grantors share in the partition of
the property owned in common. Since a co-owner is entitled to sell his undivided share, a sale
of the entire property by one co-owner without the consent of the other co-owners is not null
and void; only the rights of the co-owner/seller are transferred, thereby making the buyer a coowner of the property.
The appropriate recourse of co-owners where their consent was not secured in a sale of
the entire property as well as in a sale merely of the undivided shares of some of the co-owners
is an action for PARTITION. Neither recovery of possession nor restitution can be granted
since the defendant buyers are legitimate proprietors and possessors in joint ownership of the
common property claimed. The Deed of Sale executed by Sixto Medrano in favor of Tiburcio
Balitaan is a valid conveyance only insofar as the share of Sixto Medrano in the co-ownership is
concerned.
SALVADOR V. COURT OF APPEALS
G.R. No. 109910 April 5, 1995
Ponente: Davide Jr.
FACTS:
Alipio Yabo was the owner of Lot No. 6080 and Lot No. 6180 situated in Barrio Bulua,
Cagayan de Oro City, containing an area of 1,267 and 3,816 square meters, respectively. Title
thereto devolved upon his nine children, namely, Victoriano, Procopio, Lope, Jose, Pelagia,
Baseliza, Francisca, Maria, and Gaudencia, upon his death sometime before or during the
second world war.
On 28 April 1976, Pastor Makibalo, who is the husband of Maria Yabo, one of Alipio's
children, filed with the then Court of First Instance of Misamis Oriental a complaint, docketed
as Civil Case No. 5000, against the spouses Alberto and Elpia Yabo for "Quieting of Title,
Annulment of Documents, and Damages." In the complaint, he alleged that he owned a total of
eight shares of the subject lots, having purchased the shares of seven of Alipio's children and

inherited the share of his wife, Maria, and that except for the portion corresponding to
Gaudencia's share which he did not buy, he occupied, cultivated, and possessed continuously,
openly, peacefully, and exclusively the two parcels of land. He then prayed that he be declared
the absolute owner of 8/9 of the lots in question.
On 8 October 1976, the grandchildren and great-grandchildren of the late Alipio
Yabo lodged with the same court a complaint for partition and quieting of title with
damages, docketed as Civil Case No. 5174, against Pastor Makibalo, Enecia Cristal, and the
spouses Eulogio and Remedies Salvador. They alleged that Lot No. 6080 and Lot No. 6180 are
the common property of the heirs of Alipio Yabo, namely, the plaintiffs, defendant Enecia
Cristal, Maria Yabo and Jose Yabo, whose share had been sold to Alberto Yabo; that after
Alipio's death, the spouses Pastor and Maria Makibalo, Enecia Cristal and Jose Yabo became
the de facto administrators of the said properties; and that much to their surprise, they
discovered that the Salvador spouses, who were strangers to the family, have been harvesting
coconuts from the lots, which act as a cloud on the plaintiffs' title over the lots.
The plaintiffs then prayed, among others that the Salvador spouses be declared as
having no rights thereto except as possible assignees of their co-defendants, Pastor Makibalo
and Enecia Cristal; and that the lots be partitioned according to law among the aforementioned
co-owners.
The trial court decided in favor of the plaintiff. The Court of Appeals, on the other hand,
held that prescription and laches have not run against the private respondents with respect to
the 1/9 share of Maria Yabo in the estate of her father.

ISSUE/S: Whether or not prescription and laches can be applied against the co-heirs of Pastor
Makibalo?

HELD: No.
It has been said that Article 494 of the Civil Code which provides that each co-owner
may demand at any time the partition of the common property implies that an action to
demand partition is imprescriptible or cannot be barred by laches. 29 The imprescriptibility of
the action cannot, however, be invoked when one of the co-owners has possessed the property
as exclusive owner and for a period sufficient to acquire it by prescription.
A mere silent possession by a co-owner, his receipt of rents, fruits or profits from the
property, the erection of buildings and fences and the planting of trees thereon, and the
payment of land taxes, cannot serve as proof of exclusive ownership, if it is not borne out by
clear and convincing evidence that he exercised acts of possession which unequivocably
constituted an ouster or deprivation of the rights of the other co-owners.
Thus, in order that a co-owner's possession may be deemed adverse to the cestui que
trust or the other co-owners, the following elements must concur: (1) that he has performed
unequivocal acts of repudiation amounting to an ouster of the cestui que trust or the other coowners; (2) that such positive acts of repudiation have been made known to the cestui que
trust or the other co-owners; and (3) that the evidence thereon must be clear and convincing.

The records do not show that Pastor Makibalo adjudicated to himself the whole estate
of his wife by means of an affidavit filed with the Office of the Register of Deeds as allowed
under Section 1 Rule 74 of the Rules of Court, or that he caused the issuance of a certificate of
title in his name or the cancellation of the tax declaration in Alipio's name and the issuance of a
new one in his own name. The only act which may be deemed as a repudiation by Pastor of the
co-ownership over the lots is his filing on 28 April 1976 of an action to quiet title (Civil Case No.
5000). The period of prescription started to run only from this repudiation. However, this was
tolled when his co-heirs, the private respondents herein, instituted on 8 October 1976 an action
for partition (Civil Case No. 5174) of the lots. Hence, the adverse possession by Pastor being for
only about six months would not vest in him exclusive ownership of his wife's estate, and
absent acquisitive prescription of ownership, laches and prescription of the action for partition
will not lie in favor of Pastor.
Fangonil- Herrera v. Fangonil
G.R. No. 169356. August 28, 2007
CHICO-NAZARIO, J:
FACTS: Petitioner and respondents are children of the late Fabian and Maria Lloren Fangonil.
The spouses died intestate, leaving an estate consisting of seven parcels of land. Prior to their
death, transactions involving parcels number six and seven took place. A portion of the sixth
land and parcel seven were sold with a right to repurchase to Oribello and Estacion,
respectively. It was petitioner who repurchased and redeemed these properties in 1956 and
1959. In 1995, six of the seven children, excluding petitioner, filed a petition for judicial
partition of the seven parcels of land. Petitioner opposed claiming exclusive ownership over
parcels six and seven, and that the right to claim by the respondents had long prescribed as a
result of their inaction.
ISSUE: Whether or not petitioner is correct.
HELD: Petitioner's possession of parcels 6 and 7 did not ripen into sole and exclusive ownership
thereof. Prescription applies to adverse, open, continuous, and exclusive possession. In order
that a co-owner's possession may be deemed adverse to the other co-owners, the following
elements must concur: (1) that he has performed unequivocal acts of repudiation amounting to
an ouster of the other co-owners; (2) that such positive acts of repudiation have been made
known to the other co-owners; and (3) that the evidence thereon must be clear and convincing.
As a rule, prescription does not run in favor of a co-heir or co-owner as long as he expressly or
impliedly recognizes the co-ownership; and he cannot acquire by prescription the share of the
other co-owners, absent a clear repudiation of the co-ownership. An action to demand partition
among co-owners is imprescriptible, and each co-owner may demand at any time the partition
of the common property.
Adlawan vs Adlawan (Co-ownership)
G.R. No. 161916
January 20, 2006
Justice Consuelo Ynares-Santiago

FACTS:
A house and lot (lot 7226) was registered in the name of Dominador Adlawan, the father of
(petitioner) Arnelito Adlawan. He is the acknowledged illegitimate child of Dominador who is
claiming that he is the sole heir. He then adjudicated the said house and lot to himself and
allegedly out of generosity allowed the siblings of his father to occupy the property provided
that they vacate when asked. Time came when he demanded that they vacate and when they
refused he filed an ejectment suit against them.
His aunt and uncle on the other hand, Narcisa (70) and Emeterio (59) denied his allegations
claiming that the said property was registered in their parents name and they had been living in
the said house and lot since birth. The only reason why the title to said property was
transferred in Dominadors name was when their parents were in need of money for renovating
their house, their parents were not qualified to obtain a loan and since Dominador was the only
one who had a college education, they executed a simulated deed of sale in favor of
Dominador.
The MTC dismissed the complaint holding that Arnelitos filiation and the settlement of the
estate are conditions precedent for the accrual of the suit. And since Dominador was survived
by his wife, Graciana, her legal heirs are entitled to their share in the lot.
On appeal, the RTC reversed MTCs decision and ordered Narcisa and Emeterio to turn over the
possession of the lot to Arnelito. It also granted the motion of execution which was opposed by
the nephew and nieces of Graciana who claim that they have a share in the lot as heirs of
Graciana.
The CA reinstated the decision of the MTC holding that Arnelito and the heirs of Graciana are
co-heirs thus he cannot eject them from the property via unlawful detainer. Thus the case at
bar.
ISSUE: Whether or not Arnelito can validly maintain the ejectment suit.
HELD:
NO. The theory of succession invoked by Arnelito would prove that he is not the sole heir of
Dominador. Since Dominador was survived by his wife, upon his death, Arnelito (the son) and
Graciana (the wife) became co-owners of the lot. Upon Gracianas death, her share passed on
to her relatives by consanguinity thus making them co-owners as well.
Petitioner contends that Art. 487 allows him to file the instant petition. (Art. 487. Any one of
the co-owners may bring an action in ejectment.) It is true that a co-owner may bring such an
action without necessity of joining all the co-owners as plaintiffs because it is presumed to be
instituted for the benefit of all, BUT if the action is for the benefit of the plaintiff alone the
action should be dismissed.
Since Arnelito brought the suit in his name and for his benefit alone and his repudiation of the
ownership of the other heirs, the instant petition should be dismissed.

Title: Fernandez vs. Spouses Tarun


G.R. No. 143868
Date: November 14, 2002
Ponente: Panganiban, J.
Facts:
A fishpond situated at Dagupan City is co-owned by the brothers Antonio, Santiago, Demetria,
and Angel Fernandez, together with their uncle Armando, to the extent of 1/6 thereof. It was
subsequently increased to 1/5, on account of the 1/6 share of Armando, who died single and
without issue, which accrued in favour of five remaining co-owners. Antonio and Demetria sold
their respective shares to spouses Tarun, both registered and annotated on the OCT. Later, the
co-owners of the subject fishpond and other fishpond executed a Deed of Extra-Judicial
Partition of two parcels of registered land with exchange of shares. Among the parties to the
Deed are the Fernandez siblings. It was stipulated that the parties recognize and respect the
sale in favour of spouses Tarun. By virtue of the Deed of Extra-judicial Partition, Angel
Fernandez exchanged his share on the other fishpond, making Angel and spouses Tarun coowners of the first fishpond. A TCT was issued in the name of Angel and spouse Tarun.
However, it was Angel and later on his heirs who remained in possession of the entire fishpond.
When Angel was still alive, spouses Tarun sought the partition of the property and their share
of its income. Angel refused to heed their demand. After the death of Angel, spouses Tarun
wrote his heirs of their desire for partition but this was rejected by the latter.
Issues:
1. Whether or not petitioners are entitled to exercise their right of legal redemption.
2. Whether or not the Deed of Extra-judicial Partition is void and efficacious.
Held:
No. Under Article 1620, a co-owner may exercise the right of redemption in case the shares of
co-owner or any of them are sold to a third person. It is quite clear that respondents are
petitioners co-owners. The sale of the property to spouses Tarun had long been consummated
before petitioners succeeded their predecessor, Angel. By the time petitioners entered into coownership, respondents were no longer third persons, but had already become co-owners of
the whole property.
No. The Deed of Extra-Judicial Partition is not void and inefficacious. Petitioners are bound by
the Deed because contract not only take effect between the parties but also extend to their
assigns and heirs. The Extra-Judicial partition was more beneficial to Angel. By swapping shares
of the other fishpond with shares in the disputed fishpond, his ownership became contiguous
and compact only one fishpond, instead of being merely shared with the other co-heirs in two
different fishpond.

Title: Galvez vs Court of Appeals


G.R. No: 157954
Date: March 24, 2006
Ponente: Associate Justice MINITA V. CHICO-NAZARIO
FACTS: Ulpiano and Paz Galvez are children of Timotea F. Galvez, who died intestate. However,
Ulpiano predeceased Timotea and was survived by his son Porfirio Galvez. Timotea left a parcel
of land which passed by succession, both to Timoteas daughter, Paz Galvez, and to the
formers grandson, Porfirio, the latter succeeding by right of representation as the son of
Ulpiano. Paz Galvez executed an affidavit of adjudication stating that she is the true and lawful
owner of the said property, which said property was sold to Carlos Tam without the knowledge
and consent of Porfirio Galvez. Subsequently, Carlos Tam sold the same to Tycoon Properties,
Inc.
ISSUE/S:
1. WON respondents claim over the subject property which is based on an implied trust,
has already prescribed because the action was filed 24 years after petitioner repudiated
the said trust.
2. WON respondents claim is already barred by laches because he failed to assert his
alleged right for almost 24 years.
3. WON petitioners Carlos Tam and Tycoon Properties are buyers in good faith.
HELD:
1. No, prescription will not lie. This case is governed by the rules on co-ownership. Both
Paz and Porfirio are co-owners having inherited the same from a common ancestor.
Article 494 of the Civil Code provides that prescription shall not run in favor of a coowner or co-heir against his co-owners or co-heirs as long as he expressly or impliedly
recognizes the co-ownership. Prescription as a mode of termination a relation of coownership must have been preceded by repudiation. Repudiation is subject to certain
conditions: 1) a co-owner repudiates the co-ownership; 2) such an act of repudiation is
clearly made known to the other co-owners; 3) the evidence thereon is clear and
conclusive; and 4) he has been in possession through open, continuous, exclusive, and
notorious possession of the property for the period required by law.
2. No, the equitable remedy of laches is unavailing in this case. Laches cannot be used to
defeat justice or perpetrate fraud and injustice. Neither should its application be used to
prevent the rightful owners of a property from recovering what has been fraudulently
registered in the name of another.
3. No, a purchaser in good faith and for value is one who buys the property without notice
that some other person has right to or interest in such property and pays its fair price
before he has notice of the adverse claims and interest of another person. In the case at
bar, Tam did not exert efforts to determine the previous ownership of the property in
question and relied only on the tax declaration in the name of Paz.

Heirs of Reyes, Jr. v. Reyes


G.R. No. 158377
August 04, 2010
BERSAMIN, J
FACTS: Antonio Reyes and his wife, Leoncia Mag-isa Reyes (Leoncia), were owners of a parcel of
residential land located in Pulilan, Bulacan. The couple had four children, namely: Jose Reyes,
Sr. (Jose, Sr.), Teofilo Reyes (Teofilo), Jose Reyes, Jr. (Jose, Jr.) and Potenciana Reyes-Valenzuela
(Potenciana). Antonio Reyes died intestate, and was survived by Leoncia and their three sons,
Potenciana having predeceased her father. Potenciana also died intestate, survived by her
children. Jose, Jr., and his family resided in the house of the parents, but Teofilo constructed on
the property his own house, where he and his family resided. Leoncia and her three sons
executed a deed denominated Kasulatan ng Biling Mabibiling Muli, whereby they sold the land
and its existing improvements to the Spouses Benedicto Francia and Monica Ajoco (Spouses
Francia) for P500.00, subject to the vendors right to repurchase for the same amount sa oras
na sila'y makinabang. Potencianas heirs did not assent to that deed. Nonetheless, Teofilo and
Jose, Jr. and their respective families remained in possession of the property. Leoncia and her
children did not repay the amount of P500.00. The Spouses Francia both died intestate.
Alejandro Reyes (Alejandro) paid the obligation of Leoncia. Thus, the heirs of Spouses Francia
executed a deed entitled Pagsasa-ayos ng Pag-aari at Pagsasalin whereby they transferred and
conveyed to Alejandro all their rights and interests in the property. Alejandro executed
a Kasulatan ng Pagmeme-ari, wherein he declared that he had acquired all the rights and
interests of the heirs of the Spouses Francia, including the ownership of the property, after the
vendors had failed to repurchase within the given period. Nevertheless, Alejandro, his
grandmother (Leoncia), and his father (Jose, Sr.) executed a Magkakalakip na Salaysay, by
which Alejandro acknowledged the right of Leoncia, Jose, Jr., and Jose, Sr. to repurchase the
property at any time. Leoncia died intestate. She was survived by Jose, Sr., Teofilo, Jose, Jr. and
the heirs of Potenciana. Even after Leonicas death, Teofilo and Jose, Jr., with their respective
families, continued to reside in the property. Alejandro also died intestate. Surviving him were
his wife, Amanda Reyes, and their children, namely: Consolacion Reyes, Eugenia ReyesElvambuena, Luciana Reyes-Mendoza, Pedrito S. Reyes, Merlinda Reyes-Famodulan, Eduardo
Reyes and June S. Reyes (respondents herein). Respondent Amanda Reyes asked the heirs of
Teofilo and Jose, Jr., to vacate the property because she and her children already needed
it. After the petitioners refused to comply, she filed a complaint against the petitioners in
the barangay, seeking their eviction from the property. No amicable settlement was reached.
ISSUES:
property

1)Whether or not Alejandro became a co-owner when he redeemed the subject

2)Whether or not Alejandro and his heirs (respondents herein) acquire the mortgaged
property
through prescription?
HELD:

1. NO. When Alejandro redeemed the property, he did not thereby become a co-owner
thereof, because his father Jose, Sr. was then still alive. Alejandro merely became the
assignee of the mortgage, and the property continued to be co-owned by Leoncia and
her sons Jose, Sr., Jose Jr., and Teofilo. As an assignee of the mortgage and the mortgage
credit, Alejandro acquired only the rights of his assignors, nothing more. He himself
confirmed so in the Magkasanib na Salaysay, whereby he acknowledged the co-owners
right to redeem the property from him at any time (sa ano mang oras) for the
same redemption price of P500.00. It is worthy to note that Alejandros confirmation in
the Magkasanib na Salaysay of the co-owners right to redeem was made despite 15
years having meanwhile elapsed from the execution of the original Kasulatan ng Biling
Mabibiling Muli until the execution of the Magkasanib na Salaysay
2. NO. It is true that Alejandro became a co-owner of the property by right of
representation upon the death of his father, Jose Sr. As a co-owner, however, his
possession was like that of a trustee and was not regarded as adverse to his co-owners
but in fact beneficial to all of them. The Court cannot accept the respondents
contention that Alejandro, having earlier repudiated the co-ownership, acquired
ownership of the property through prescription. In order that a co-owners possession
may be deemed adverse to that of the cestui que trust or the other co-owners, the
following elements must concur:
1. The co-owner has performed unequivocal acts of repudiation of the coownership amounting to an ouster of the cestui que trust or the other coowners;
2.

Such positive acts of repudiation have been made known to the cestui que
trust or the other co-owners;

3.

The evidence on the repudiation is clear and conclusive; and

4.

His possession is open, continuous, exclusive, and notorious.

The concurrence of the foregoing elements was not established herein. For one,
Alejandro did not have adverse and exclusive possession of the property, as, in fact, the
other co-owners had continued to possess it, with Alejandro and his heirs occupying
only a portion of it. Neither did the cancellation of the previous tax declarations in the
name of Leoncia, the previous co-owner, and the issuance of a new one in Alejandros
name, and Alejandros payment of the realty taxes constitute repudiation of the coownership. The sole fact of a co-owner declaring the land in question in his name for
taxation purposes and paying the land taxes did not constitute an unequivocal act of
repudiation amounting to an ouster of the other co-owner and could not constitute
adverse possession as basis for title by prescription

EDCA Publishing and Distribution Corp. v. Santos Bookstore


FACTS: A person identifying himself as Professor Cruz placed an order by telephone with
EDCA Publishing and Distribution Corporation for 406 books, payable on delivery. EDCA
prepared the sales invoice and delivered the books as ordered, for which Cruz issued a personal
check. Cruz then sold the books to Leonor Santos. After verifying the sellers ownership through
the invoice receipts, Santos paid him P1,700. Cruz made a second order from EDCA. Upon
discovering that the first check had not cleared, EDCA called up De La Salle University where
Cruz claimed to be dean to verify his identity and was informed that there was no such person
under its employ. EDCA had Cruz arrested and found that his real name was Tomas de la Pena.
EDCA then went to Santos Bookstore, made threats to the private respondents herein for
buying stolen property and through force, seized 120 books without a warrant.
ISSUE: Whether or not EDCA was unlawfully deprived within the meaning of Article 559 of the
civil code, of the said property because the check issued by the imposter was dishonored.
HELD: No, EDCA was not. The Supreme Court held that the case does not fall under the purview
of Art. 559. The contract of sale in this case was consensual, and under 1475 was perfected at
the moment there was meeting of the minds upon the thing and the price. Ownership is
transferred upon delivery. To rule that ownership shall not pass until full payment only occurs if
there is a stipulation to that effect. Non-payment only creates a right to demand payment.
There was no unlawful deprivation in this case. The fraud and deceit practiced by de la Pena
armarks this sale as a voidable contract (Article 1390 N.C.C.). Being a voidable contract, it is
susceptible of either ratification or annulment. If the contract is ratified, the action to annul it is
extinguished (Article 1392, N.C.C.) and the contract is cleansed from all its defects (Article 1396,
N.C.C.); if the contract is annulled, the contracting parties are restored to their respective
situations before the contract and mutual restitution follows as a consequence (Article 1398,
N.C.C.). However, as long as no action is taken by the party entitled, either that of annulment or
of ratification, the contract of sale remains valid and binding.
Actual delivery of the books having been made, Cruz acquired ownership over the books
which he could then validly transfer to the private respondents. The fact that he had not yet
paid for them to EDCA was a matter between him and EDCA and did not impair the title
acquired by the private respondents to the books.

LEDESMA VS. CA
GR no. 86051

FACTS:
Citiwide sold 2 cars to a certain Jojo Consunji evidenced by 2 invoices. Upon delivery of the cars,
Jojo paid with a Managers Check (PhP101,000.00). When Citiwide deposited the check, it was
dishonored for being tampered. Amount was changed from 101.00 to 101,000.00.

Citiwide reported the crime to the Phil. Constabulary where he found that Consunji was actually
Armando Suarez, a professional criminal.

One car was found abandoned, while the other was discovered to be in the possession of Jaime
Ledesma, who claims to have purchased the car in good faith from the registered owner,
evidenced by the LTO Registration.

RTC ordered the car to remain in the possession of Ledesma.

CA overruled RTC, stating that Citiwide was unlawfully deprived of property through false
pretenses amounting to fraud.

ISSUE:
Whether or not the owner was unlawfully deprived of the property?

HELD:
No. There was a perfected unconditional contract of sale. Failure to pay by Suarez through the
subsequent dishonor of the check did not render the contract of sale void. Ownership was
already transferred by the delivery of the cars to Suarez. Petitioner is a buyer in good faith and
therefore, article 559 will not apply.

Spouses Suobiron vs. Court of Appeals


GR No. 109903
November 20, 1995
Ponente: Bellosillo, J.
On December 2, petitioner spouses Andres and Socorro Suobiron, Jose Sullano Jr. and Ireneo
Ferraris instituted an action to annul the the orders of the CFI of Iloilo.
Petitioners alleged that the land registration court acted without or in excess of jurisdiction
from issuing both orders because of the requirements of the law on reconstitution of court
records were not complied with thus rendering void not only the orders but also the decrees
and certificates of the title issued thereunder.
Private respondents denied the allegations for the annulment of the orders and decrees. They
counterclaimed for the delivery of the property in litigation consisting of 26.5 hectares of
sugarland and for the net produce which they could have received had they not been deprived
of possession thereof.
Taking advantage in the meantime of the chaotic conditions during the war, Quintin and
Bernabe Lorezo entered the litigated property and appropriated the produce thereof to the
damage and prejudice of the registered owners.
Consequently, on August 26, 1947, the Adelantars filed an action in the CFI of Iloilo against the
Lorezos for recovery of possession.
The property soon became the subject of a cadastral survey. Fortunata Ponce, who was already
a widow, filed an answer claiming ownership. The spouses Suobiron also filed an answer
claiming ownership of portions thereof thus promoting the cadastral court to advise the parties
to file the proper action and to litigate the question of ownership.
Issue:
Whether or not herein Spouses petitioners are the rightful owners of the portions of the
subject property
Held:
The decision in declaring the Adelantar spouses the owners of the two parcels of land claimed
by Basilia Lorezo, Isabel Lorezo, and Canuto Lucero is conclusive upon the parties therein as
well as their successors in interest, the parties herein, under the doctrine of res judicata.
Citing the case of Rodriguez v. Francisco, the SC held that "... on the date of service of summons
upon the appellees in this case, considering that appellant was thereafter declared, owner by

final judgment, the appellees possession in good faith was interrupted and hence from that
time, he lost the right to the fruits."
In turn, that decision was based on Tacas v. Tobon where the Supreme Court, citing Manresa,
stated
But to every possessor in good faith there comes a time when he is considered a possessor in
bad faith. When the owner or possessor with a better right comes along, when he becomes
aware that what he had taken for granted is at least doubtful, and when he learns the grounds
in support of the adverse contention, good faith ceases. The possessor may still believe that his
right is more secure, because we resign ourselves with difficulty to the sight of our vanishing
hopes; but when the final judgment of the court deprives him of the possession, all illusion
necessarily disappears. Although he may not have been convinced of it before, the possessor
becomes aware that his possession is unlawful from the time he learns of the complaint, from
the time he is summoned to the trial. It is at this time that his possession is interrupted,
according to article 1945, and that he ceases to receive the fruits, according to the first
paragraph of article 451..."

Вам также может понравиться